SlideShare a Scribd company logo
1 of 29
MDCU COMPREHENSIVE EXAM                    4. stretching of quadriceps muscle
              STEP 2                          5. increase activity with knee
        27 กุมภาพันธ 2552                          flexion >90 degree
                                          4. A man experienced a low back pain
   ขอบคุณเพื่อนๆที่ชวยกันจดจํา
                       
                                          after lifting object from the floor. He was
 ขอบคุณเกดกับพี่จุบที่ชวยเปนเสมียน
                         
                                          diagnosed with protusion of L4-5 disc.
                                          Which of the following nerve root will be
ORTHO                                     compressed and what symptoms will he
1. What is the mechanism of injury from   present with?
seat-belt injury?                             1. L4 nerve root – weakness of
    1. Flexion injury                               quadriceps muscle
    2. Flexion rotation injury                2. L4 nerve root – weakness of
    3. Extension injury
                                                    tibialis anterior
    4. Extension distraction injury
                                              3. L5 nerve root – sensory loss at
    5. Vertical compression injury
2. What is INCORRECT about flexion                  heel
rotation injury of TLS spine?                 4. L5 nerve root – weakness of
    1. It’s unstable injury.                        tibialis anterior
    2. Neurological involvement is            5. L5 nerve root – decreased ankle
        common.                                     jerk
    3. It cannot be treated
                                          5. A 25 yr-old man, sudden right back
        conservatively.
    4. Surgery is indicated when there    pain radiate to Rt. leg while he was
        is progressive neurological       picking up object from floor. What’s not
        deficit.                          appropriate for acute management?
    5. Bad prognosis in the presence          1. bed rest
        of abnormal reflexes.                 2. NSAIDs
3. In the case of chronic OA knee. What       3. pelvic traction
is NOT appropriate management?                4. lumbosacral support
    1. paracetamol                            5. transcutaneous electrical nerve
    2. ultrasound therapy                           stimulation
    3. stretching of hamstring muscle
6. Postpartum woman in breast-feeding            1.   ultrasound therapy
period presented with pain and swelling          2.   moist heat
of Rt. wrist. PE: tender, nodule at radial       3.   laser
side of Rt. hand. The pain was                   4.   cold pack
aggravated by doing ulnar deviation.             5.   hydrotherapy
What is the diagnosis?
                                              9. เด็กชายอายุ 5 ป มีอาการเจ็บสะโพก
    1. carpal tunnel syndrome                 ขวามา 3 เดือน เริ่มเดินกะเผลกมา 1 เดือน
    2. cubital tunnel syndrome                ตรวจรางกาย hip joint มี limit ROM.ขอใด
    3. carpal ganglion                        ตอไปนี้ผิด
    4. stenosing tenosynovitis                     1. โรคดังกลาวเกิดจาก        ischemic
    5. rheumatoid         arthritis    with            necrosis ของ femoral head
                                                   2. เปนในเด็กชายมากกวาเด็กหญิง
          rheumatoid nodule
                                                   3. พบ unilateral มากกวา bilateral
7. ผูปวยหญิงวัยรุน ปวดขอมือขวา, 2nd Rt.
                                                   4. prognosis จะแยกวาถามีอาการ
PIP และ Lt. elbow มา 1 สัปดาห ปฏิเสธ                  ตอน 10 ป
ประวัติ Sexual intercourse PV: normal              5. การรักษาทําเพื่อปองกัน deformity
ตรวจรางกาย warm, swelling and                10. เด็ก 12 ป มีอาการ severe pain right
tenderness at 2nd PIP and Rt. wrist, Lt.      thigh, มารดาใหประวัติวา 1 สัปดาหกอน
elbow. What is the most appropriate           เกิดอุบัติเหตุจากการเลนฟุตบอล        ตรวจ
                                              รางกายไมมีไข เด็กมีอาการเจ็บที่ตนขาขวา
                                                                                 
investigation?
                                              ไมยอมขยับขา จะสงตรวจอะไรเพื่อชวยการ
    1. Lt. elbow arthrocentesis               วินิจฉัย
    2. Rt. wrist arthrocentesis                    1. ultrasound Rt thigh
    3. H/C                                         2. X-ray hip with Rt thigh
    4. cervical swab                          ER MED AND TRAUMA
8. ผูปวยหญิง 70 ป chronic bilateral knee
                                             11. Which of the following management
pain with morning stiffness less than 20      can offer the best chance in treatment
                                              of PEA?
min for 6 months, PE: bow leg, atrophy
                                                   1. Early defibrillation
of quadriceps muscle, tender at medial             2. Early administration of atropine
side of both knee, full ROM, pain on               3. Early transcutaneous pacing
end. What is the most appropriate
management?
4. Rapid         identification    and        3. observe for signs of
        correction       of       immediate            compartment syndrome
        correctable cause                         4. tetanus toxoid IM and tetanus
12. 30 yr-old man sustained stab chest                 antitoxin
wound at Lt. parasternal border. He was       15. Which is CORRECT about brain
restless but can speak fluently. BP           edema?
80/60 PR 110/min PE: engorged neck                1. Brain edema is solely due to
vein, equal breath sound, no tracheal                  vasogenic edema.
deviation. After resuscitation with               2. Blood brain barrier disruption is
Oxygen and RLS, what’s the next                        the least likely to cause brain
management?                                            edema
    1. ET           intubation         with       3. Uptake and degradation of fluid
        hyperventilation                               protein by glial cells is one of
    2. continue RLS IV                                 the mechanism of vasogenic
    3. consider pericardiocentesis and                 edema.
        refer to trauma center                    4. Mannitol is effective only in the
    4. needle thoracocentesis                          area that the BBB is broken.
13. A man was struck by a lightning 30        16. ผูปวยหญิงอายุ 20 ปมาหองฉุกเฉิน
minutes ago. PE reveals burned wound          ดวยหอบ เกร็ง หายใจเร็ว มือจีบ ถาทาน
at Lt. buttock and Rt. arm, otherwise         เปนแพทยเวร จะทําอยางไร
WNL. He is clinically stable. What is the         1. ปลอบผูปวยใหใจเย็นๆแลวซัก
                                                                 
next management?                                       ประวัติ
    1. Admit for observation                      2. เอาถุงกระดาษครอบปากและจมูก
    2. colostomy                                       ทันที
    3. sigmoidoscopy                              3. ให valium 10 mg IV ทันที
    4. wound dressing and discharge               4. เจาะ blood gas ทันที
14. A man was bitten by a snake at his        17. ผูปวยประสบอุบติเหตุรถชน มี pelvic
                                                                    ั
leg 30 minutes ago. The leg is swollen        fracture (APC III) with perineal
and erythematous. There is a fang mark        laceration. After initial resuscitation, the
at the wound. He also has hematuria.          subsequent management would consist
Which of the following management is          of which of the followings?
NOT appropriate?                                  1. ORIF with debridement and
    1. coagulogram                                     colostomy
    2. antivenum
2. pelvic       stabilization   and         3. fiberoptic guided nasotracheal
          colostomy                                  intubation
      3. ORIF and debridement                     4. cricothyroidotomy
      4. explor perineum                          5. tracheostomy
      5. pelvic traction and debridement    21. ผูปวยอายุ 25 ปประสบอุบตเหตุ     ั ิ
18. ผูปวยชายมีอาการปวดทองนอยรุนแรง      รถยนต แรกรับที่ ER ขาสองขางไมเทากัน
เปนตอนที่กําลังปสสาวะ แรกๆปสสาวะไหล      ผูปวยหายใจไดเองปกติ เสียงลมหายใจ
ดี จากนั้นปสสาวะหยุดทันที และมีอาการ       เทากัน 2 ขาง BP 90/60 mmHg ให RLS
ปวดขึ้นมาทันที คิดถึงโรคใดมากที่สุด         IV load ไป 2000 ml ปรากฏวา BP ยังคง
      1. urethral stone                     90/60 อยู ควรทําอยางไรตอไป
      2. vesical stone                            1. load RLS ตออีก 2000 ml.
      3. ureteral stone                           2. emergency explor. Lap.
      4. acute pyelonephritis                     3. ทํา DPL ที่ lower abdomen
      5. acute cystitis                           4. emergency angiography
19. ผูปวยอายุ 18 ปถกสุนัขกัดมา 1 เดือน
                        ู                         5. group O Rh - blood transfusion
มีประวัตฉีดบาดทะยักครบ 5 เข็มแลว และ
           ิ                                22. ผูปวยตกตึก 3 ชั้น unconscious ลอง
เคยฉีด Rabies vaccine ครบ 5 เข็มมาเมื่อ     กดที่ supraorbital ridge ผูปวยไมลืมตา ไม
อายุ 8 ป จะให vaccine อยางไร             พูด จงบอก GCS
      1. Rabies vaccine booster 2 doses           1. 4
      2. Rabies vaccine 3 doses                   2. 5
      3. Rabies vaccine 5 doses                   3. 6
      4. Rabies vaccine 5 doses and               4. 7
          Rabies immunoglobulin                   5. 8
      5. เจาะเลือดตรวจระดับ Rabies Ab       23. ผูหญิงวัยรุน นําสง ER เนื่องจากหมด
          ถามากกวา protective level ไม   สติ ตรวจรางกายมี pinpoint pupil หายใจ
          ตองให vaccine                   ชา ความดันต่ํา ไมมี neurodeficit คิดวา
20. ผูปวยชาย โดนรถชนหมดสติ มีรอยช้ํา      เกิดจากอะไร
ที่ทายทอยและแผลที่คอดานหลัง ที่ ER มี           1. opioid overdose
inspiratory stridor ควรจัดการกับ airway           2. alcohol intoxication
ของผูปวยรายนี้อยางไร                           3. benzodiazepine overdose
      1. head tilt-jaw thrust และ ใช       PED
          AMBU bag                          24. A 10-yr old girl visits you because of
      2. direct laryngoscope and ET         a goiter. Her mother states that she has
          intubation                        poor performance at school. PE: diffuse
thyroid       enlargement,        rubbery        2. Serum electrolyte
consistency, irregular surface. TFT:             3. USG KUB system
FT4=6 FT3=1.4 TSH=20. What is the                4. VCUG
best investigation for diagnosis?
                                                 5. IVP
    1. thyroid globulin
    2. USG thyroid                          27. 5 yr-old boy was stung by wasps 10
    3. FNA                                  days ago. He came to hospital because
    4. thyroid scan                         of fever, swollen wrist and ankle and
    5. Antinuclear antibody                 urticaria. What is the most likely
25. Male term neonate with exclusive        diagnosis?
breast feeding developed jaundice on             1. serum sickness
DOL 3. He was treated with intensive
                                                 2. post infective arthritis
phototherapy. Serum bilirubin decline
from 20 mg/dl to 19 mg/dl in 4 hr. His           3. pyomyositis
laboratory test shows that Coomb’s test          4. toxic vasculitis
negative, G6PD not deficient. PBS:               5. Henoch-Schonlein purpura
microspherocyte. Maternal blood group       28. Male neonate, BW 2000g with
is B Rh+. His blood group is O Rh+.         dyspnea and cyanosis. PE: meconium
What is the next management?
                                            staining at nail, grunting and stridor.
    1. continue             phototherapy,
        exchange if MB>20                   What is the first management?
    2. continue             phototherapy,        1. give oxygen
        exchange if MB>25                        2. suction at hypopharynx
    3. partial exchange transfusion              3. evaluate APGAR
    4. total exchange transfusion                4. Intubation
    5. stop breast feeding and switch            5. positive pressure ventilation
        to formula, then repeat MB
                                            29. A 1-hr old male neonate with BW
        again
26. A full term neonate, PE: penoscrotal    2000 g. BT 35.5°C HR 180/min BP
opening of the urethral meatus, cannot      40/28 mmHg. ABG: pH=7.16 PaO2=30
palpate both sides of testis in the         PCO2=50 HCO3=12. What is the least
scrotum. What is the emergency              likely to be the problem in this patient?
management?                                      1. cold stress
    1. Serum creatinine
                                                 2. respiratory acidosis
3. metabolic acidosis                        3. salbutamol NB with ipatropium
    4. hypoxemia                                     bromide
    5. hypovolemic shock                         4. budesonide NB
                                                 5. adrenaline NB
30. An 8-month-old girl presents with 2
                                             33. 2 yr-old male presented with calf
days of high fever. Physical examination     pain. PE: hematoma at calf with
reveals BT 38oC, tense anterior              tenderness, multiple large ecchymosis
fontanelle, clear sensorium, and             at lower extremities. Coagulogram
generalized maculopapular rash. Which        reveals prolonged PT, normal PTT and
of the following is the most likely          TT. What is the proper management?
diagnosis?                                       1. cryoprecipiate
                                                 2. Vit. K 10 mg IV
    1. Varicella
                                                 3. DDAVP
    2. Measles                                   4. plt. transfusion
    3. Exanthem subitum                      34. Preterm neonate GA 32 wk BW
    4. Erythema infectiosum                  1600 develops grunting and retraction
    5. Viral meningitis                      after delivery. What is the appropriate
                                             early management?
31. A 6 yr-old boy presented with                1. ABG
erythematous vesicle, pruritus and               2. CPAP
excoriation at popliteal and cubital area.       3. CXR
What is the appropriate management?              4. TPN
    1. topical cotrimazole                       5. IV fluid
    2. topical triamcinolone                 35. 6 yr-old boy presented with fever,
    3. oral doxycycline                      barking cough and stridor. PE: stridor,
32. A child presented with acute             barking cough, harsh breath sound,
asthmatic attack. At ER, he was given 3      inspiratory stridor. Subcostal and
doses of NB salbutamol. After                supracostal retraction RR 32/min What’s
administration of 3 doses, wheezing and      the appropriate management?
retraction still persist. He still has           1. salbutamol NB
respiratory distress. What is the next           2. ipatropium NB
management?                                      3. adrenaline NB
    1. IV hydrocortisone                         4. IV corticosteroid
    2. Intubation                                5. ET intubation
36. A 9 yr-old girl was brought to ER             5. primary immune deficiency
because of high fever, hematemesis           39. A male neonate, BW 2000g
and abdominal pain. PE reveals BT            developed dyspnea shortly after delivery.
39.5°C and hepatomegaly. CBC: Hct            PE: meconium staining at nail,
45.6% WBC 1500 (N 30% L 65% AL               intercostal retraction. What’s the
5%). What is the most likely diagnosis?      diagnosis?
    1. Septic shock                               1. respiratory distress syndrome
    2. Dengue hemorrhagic fever                   2. meconium aspiration syndrome
    3. Viral myocarditis                          3. transient tachypnea of newborn
    4. Aplastic anemia                       40. A 8-yr old boy presented with fever
    5. Idiopathic       thrombocytopenic     and Lt. hemiparesis for 1 wk. BT 39°C
        purpura                              with clubbing fingers. Motor weakness
37. What is the most likely to be born to    gr.II Lt. side. What’s the treatment?
mother with Graves’ disease that is               1. IV antibiotics then observe
currently controlled?                             2. intrathecal antibiotics then
                                                       observe
    1. hypothyroid infant
                                                  3. IV steroid then observe
    2. hyperthyroid infant                        4. surgical drainage
    3. mongoloid infant                           5. Emergency craniotomy
    4. infertile infant                      41. เด็กอายุ 7 ป มีไข ปวดเขามา 2
    5. infant with ambiguous genitalia       สัปดาห ตรวจรางกาย HR 120/min RR
                                             20/min BP 100/80 BT 38°C CVS:
38. A 5 yr-old boy was brought to the        pansystolic murmur gr. II/IV MS:
hospital. He has a history of recurrent      swelling Rt. knee. What’s the
respiratory tract infection and failure to   diagnosis?
thrive. Physical examination reveals              1. Rheumatic fever
chronically ill child, oral thrush and            2. Viral myocarditis
insect-bite dermatitis both legs. The             3. Kawasaki disease
other history is that both of his parents         4. Rheumatoid arthritis
died from pulmonary TB. What’s the                5. Salmonella infective endocarditis
most likely diagnosis?                       42. A 6 yr-old leukemic boy มีไขมา 5 วัน
     1. malnutrition
                                             และ bleeding per gum มี purpura ที่ขา
     2. disseminated TB
     3. disseminated candidiasis             BP 70/40 BT 40°C Hct 20% WBC 800
     4. pediatric HIV
(N5% L 95%) plt. 30000 นึกถึงภาวะใด               2. H2 blocker
มากที่สุด                                         3. UGI study
    1. septic shock                               4. USG abdomen
    2. DHF                                        5. Endoscope
    3. viral meningitis                      46. เด็กกัมพูชา 10 ขวบ มาดวยไข เจ็บคอ
    4. infective endocarditis                ไอ ตรวจรางกายมี dirty patch ที่ tonsil ทั้ง
43. เด็ก 8 ขวบ มีไข ไอ เจ็บคอ หายใจ         สองขาง ตับมามไมโต จงใหการวินิจฉัย
เหนื่อย ตรวจรางกาย BT 40°C , whitish             1. Diphtheria infection
mucopurulent patch at pharyngotonsillar           2. Infectious mononucleosis
area both sides, neck swelling,                   3. Streptococcal pharyngitis
hepatomegaly จงใหการวินิจฉัย                     4. Leptospirosis
                                             47. เด็กอายุ 2 ป มาดวย multiple flaccid
    1. Diphtheria infection
    2. Infectious mononucleosis              bullae and superficial erosion at upper
    3. Streptococcal pharyngitis             lip and chin. ควรตรวจอะไรตอเพื่อการ
    4. Leptospirosis                         วินิจฉัย
    5. retropharyngeal abscess                    1. HIV Ab
44. เด็กหญิงอายุ 3 ป ซีดมา 1 เดือน แมให        2. Tzanck smear
ประวัติวาติดจุกนม กินแตนม ไมยอมกิน
                                                  3. biopsy
ขาว PBS: hypochromic microcytic
                                                  4. KOH
anemia จงให management ที่ประหยัด
และปลอดภัยที่สุด                                  5. gram stain
    1. ให FeSO4                             48. เด็กชายอายุ 6 ป เหลืองมา 3 วัน ซึม
    2. Hb typing
                                             ลงมา 1 วัน BT 38°C BP ~90mmHg RR
    3. เจาะ serum ferritin
                                             20, stupor, hyperreflexia, marked
    4. stool occult blood                    jaundice, no hepatosplenomegaly. What
45. เด็กอายุ 10 ป มาดวยปวดทองมา 2         is the proper management?
เดือน wt loss 1 kg จะปวดมากหลังกินขาว            1. lactulose
มี awakening pain, N/V ไปพบแพทยได               2. NAC via NG tube
antacid, ยาแกปวด อาการไมดีขึ้น จงให            3. D10W 10 cc/kg IV
การรักษา                                          4. intubation and hyperventilation
    1. amitryptiline hs                      49. เด็กอายุ 1 ขวบ full term BW 3200 g
                                             ประวัติแข็งแรงมาตลอด กินเฉพาะ full cow
milk 8 oz. 20-30 cc/feed ปริมาณ 6           52. ผูที่แพไขชนิด anaphylaxis ไมควรฉีด
feeds/day ไมยอมกินอาหารอื่นๆ ตรวจ          วัคซีนปองกันโรคใดตอไปนี้
รางกายพบวาซีด ไมเหลือง ตับมามไมโต          1. JE
การเจริญเติบโตปกติดี PBS: hypochromic           2. Influenza
microcytic anemia anisocytosis 1+ CBC:          3. H. influenzae
Hb 8 MCV 67 RDW 19 other WNL.                   4. meningococcal meningitis
What is the diagnosis?                          5. pneumococcus
     1. Iron deficiency anemia              53. ผูปวยเด็กแรกคลอด ตรวจรางกาย
     2. Folate deficiency                   พบวามีลําไสมากองอยูนอกหนาทอง ไมมี
     3. B12 deficiency                      ถุงหุม ขอใดกลาวผิดเกี่ยวกับภาวะดังกลาว
     4. Niacin deficiency                       1. พบ intestinal atresia ได 10%
     5. Vit. C deficiency                             ของ case
50. ทารกแรกเกิดเพศชาย ไมมี urine ออก           2. สัมพันธกับภาวะ polyhydramnios
เลยหลังเกิด 48 ชม. ตรวจรางกายเจอ               3. พบวามี associated anomaly ได
bilateral flank masses with tense ascites             มาก
ทํา urine cath ไมได urine เลย จงใหการ        4. สายสะดือเกาะปกติ
วินิจฉัย                                    54. เด็กอายุ 5 ป มาดวย recurrent UTI,
     1. UPJ obstruction                     VCUG ปรกติ ควรสง investigation ใดตอ
     2. VUR                                     1. IVP
     3. cystic nephroma                         2. CT scan
     4. Wilm’s tumor                            3. USG
     5. neuroblastoma                       55. ทารกแรกคลอดครบกําหนด BW 3200
51. เด็กชายอายุ 15 ป ญาตินําสงเนื่องจาก   g คลอดทางชองคลอด ไมมีอาการผิดปกติ
นอนหมดสติ ขางตัวมีขวดยาเปลาตกอยู ที่     แตเจาะน้ําตาลได 35 mg/dl ควรทําอยางไร
ER no response, GCS 4 bilateral                 1. ให IV ทาง umbilical artery
pinpoint pupils RR7/min After intubation,       2. ให IV เปน 10%D/W
what is the next appropriate                    3. ใหกิน 5%D/W ทันที
management?                                     4. ให IV. . . .
     1. IV naloxone                         56. เด็กชายอายุ 3 ขวบ มาดวย cough
     2. IV flumazenil                       and dyspnea หลังจากมีอาการ URI เมื่อ 3
     3. IV atropine                         วันกอน มี respiratory distress มา 2 วัน
     4. gastric lavage                      ตรวจรางกาย cyanosis, barking cough,
     5. activated charcoal via NG tube      decreased breath sound both lungs.
                                            What’s the next management?
1. ipatropium bromide NB                   3. Electrical stimulation for Rt. limb
     2. steroid NB                              4. ADL training
     3. intubation                              5. Ambulation training
     4. salbutamol NB                       60. A man suffered from Lt. hemiplegia
57. เด็ก 8 ป แมพามารพ.เนื่องจากพบวา      applied for a rehabilitation program.
ซึมลง พบขวดเปลาไมมฉลากตกอยูขางตัว
                        ี                   After hard work of rehabilitation, he
PE: lethargy, pinpoint pupil. Oral and      develops Lt. arm edema. What is the
nasa cavity: copious secretion. lung:       appropriate management?
crepitation both lungs. Abdomen:                1. Continued rehabilitation
hyperactive bowel sound. จงใหการ               2. Elastic bandage and rest
วินิจฉัย                                        3. Consult radiologist for Doppler
     1. opioid overdose
                                                     ultrasound
     2. organophosphate poisoning
     3. Phenobarbital overdose                  4. Stellate ganglion block
     4. paraquat poisoning                  61. ผูปวยหญิง อายุ 30 ป ปวดคอและ
58. ผูปวยเด็กอายุ 6 เดือน มี underlying   สะบักทั้งสองขาง มี muscle spasm ตรวจ
เปน truncus arteriosus มาดวย mild         รางกายเจอ        trigger    point       at
cyanosis, tachypnea, tachycardia ตรวจ       supraspinatus muscle จะแนะนําการออก
รางกาย crepitation and hepatomegaly        กําลังกายอยางไร
CXR: cardiomegaly with increased
                                                     1. aerobic exercise
pulmonary vasculature What is the
appropriate Management?                              2. strengthening exercise
     1. knee-chest position                          3. stretching exercise
     2. Furosemide
                                            62. ผูหญิง 25 ป ประสบอุบัติเหตุ MCA
     3. Morphine
                                            มา 2 เดือนกอน มี spinal cord injury
     4. Bicarbonate
                                            ระดับ T2 ASIA II ควรวางแผน
     5. Beta-blocker
                                            rehabilitation program สําหรับ respiratory
REHAB
                                            system อยางไร
59. What’s the most appropriate
                                                1. Aerobic exercise
rehabilitation program for Rt. thalamic
                                                2. Diaphragmatic respiration and
hemorrhage patient diagnosed for 2
                                                     purse lip breathing
days?
                                                3. incentive spirometry
     1. Bed positioning
                                                4. Arm swing
     2. Active ROM exercise
                                            PREVENTIVE
63. ผูปวยชาย 38 ป มีประวัตครอบครัว
                                   ิ            4. ปศุสัตว
บิดาเปนเสนเลือดหัวใจขาดเลือดเมื่ออายุ
50 ป มาตรวจรางกายประจําป ควรจะสง         67. Influena vaccine ตองใหภายในกี่วัน
ตรวจอะไรมากที่สุด                            กอนการสัมผัสเชื้อจึงจะมีภูมิคมกัน
                                                                            ุ
    1. CPK                                       1. 3 วัน
    2. echocardiogram                            2. 7 วัน
    3. Serum lipid profile                       3. 14 วัน
    4. EKG                                       4. 21 วัน
    5. Exercise stress test                      5. 28 วัน
64. ผูหญิงอายุ 50 ป ไมมีอาการผิดปกติ
                                             68. หากตองการตรวจสอบการระบาดของ
มาตรวจรางกาย การตรวจหรือสงตรวจ lab
                                             polio ในชุมชน ควรบอกใหทีมสํารวจ
อะไรตอไปนี้เหมาะสมนอยที่สุด
    1. การตรวจเตานมดวยตนเอง                ตรวจสอบอะไร
    2. การตรวจทางทวารหนัก                        1. ดูแหลงน้ํา
    3. pap smear                                 2. อาหารที่ปนเปอนเชื้อ
    4. serum cholesterol                         3. สัตวเลี้ยง
    5. tumor marker                              4. เสื้อผา
65. ขอใดถูกเปนสิ่งที่ “นาทํา” ในการตรวจ
                                             69. การตรวจ stool occult blood สําหรับ
คัดกรองในประชากรกลุมเสียง   ่
    1. Chronic hepatitis infection –         คัดกรองโรคมะเร็งลําไสใหญมี sensitivity
        ตรวจ ultrasound liver                80% specificity 20% ถาในประชากรมี
    2. Chronic hepatitis infection –         ความชุกของโรค 10% โอกาสที่จะเปน
        ตรวจ AFP                             มะเร็งลําไสใหญเมื่อผลตรวจบวกเปนเทาไร
    3. มี risk atherosclerosis – ตรวจ            1. 10%
        HDL, cholesterol, TG                     2. 31%
    4. Menopause – ตรวจ bone                     3. 50%
        density                                  4. 70%
                                                 5. 90%
66. ถามี Hepatitis A ระบาดในชุมชน ควร
จะให vaccine ปองกันคนกลุมใด
    1. ครูโรงเรียนประถม
    2. อาสาสมัครสาธารณสุข
    3. บุคลากรทางการแพทย
STAT                                            72. ฝการศึกษาหาความสัมพันธของการ
70. นาย A ตองการศึกษาภาวะภูมิแพใน             บริโภค caffeine กับการเกิด migraine
กลุมเกษตรกร ไดผลดังนี้                        ไดผลดังตาราง จงหา ODD ratio
             จํานวนที่      เปนภูมิแพคดเปน
                                        ิ                         Migraine         No
             ศึกษา          (%)                                   disease          migraine
ฟารม        1000           27%                 Caffeine - 40                      200
เห็ด                                            Caffeine + 60                      100
ทํานา        6000           10%                      1. 0.2
ฟารม        3000           13%                      2. 0.33
เปด                                                 3. 0.6
นาย A สรุปวาการทําฟารมเห็ด เปนปจจัย              4. 2
เสี่ยงตอการเกิดโรคภูมิแพ จงวิเคราะหผล             5. 3
การศึกษา                                        PSYCHI
     1. ถูกตอง เพราะ % คนเปนภูมิแพใน         73. ผูปวยหญิง ถูกวิงราวและทําราย
                                                                                ่
         อาชีพฟารมเห็ดมีคามากกวา             รางกายเมื่อ 2 เดือนกอน ตอจากนั้นมี
         คาเฉลี่ย                              อาการกลัว ฝนถึงบอยๆ ไมกลาไปขางนอก
     2. ไมถูก เพราะเปน cross sectional        ตอนกลางคืน มีอาการ panic attack เปน
         study                                  บางครั้ง การวินิจฉัยตาม axis I คืออะไร
     3. ไมถูก เพราะจํานวนคนของฟารม                 1. Acute stress disorder
         เห็ดนอยกวาฟารมเปด                       2. Adjustment disorder
     4. ไมถูก เพราะจํานวนคนในแตละ                  3. Generalized anxiety disorder
         กลุมไมเทากัน                             4. Posttraumatic stress disorder
71. Incidence ของการเกิด CA lung ใน                  5. Panic disorder
smoker มีคาเทากับ 6/1000 สวน                 74. เด็กอายุ 2 ขวบโดนขัดใจ งอแง ลงไป
incidence ในกลุม nonsmoker มีคาเทากับ        ดิ้นกับพื้น              แพทยควรใหคําแนะนํา
2/1000 จงหา incidence ของการเกิด CA             ผูปกครองวาอยางไร
lung ที่เปนผลจาก smoking เทานั้น                   1. อยาขัดใจ เพราะเด็กอาจอารมณ
     a. 4%                                                เสีย
     b. 25%                                          2. ปลอยไวเฉยๆไมสนใจ ใหหยุดรอง
     c. 33%                                               เอง
                                                     3. ตีใหเจ็บ จะไดไมทําอีก
     d. 50%
                                                     4. ขูวาถาไมหยุดรอง แมจะไมรัก
     e. 67%                                          5. อธิบายเหตุผลใหเด็กจนเขาใจ แลว
                                                          ใหสิ่งที่เด็กตองการ
75. ผูปวยหญิงอาชีพครู มาพบแพทยดวย             3. เด็กไมไดสูบบุหรี่จากการติด
เรื่องมีอาการวิตกกังวลอยางมาก กังวลเรื่อง            นิโคติน
อันตรายนอกบาน กังวลเกี่ยวกับเรื่องลูก       79. ผูชายรูสกกลัวเวลาอยูที่สง กลัวจะโดด
                                                           ึ              ู
และสามี เรื่องความปลอดภัยภายในบาน
                                             ตึก เวลาเห็นมีด ก็คิดที่จะนํามีดมาแทง
จงใหการวินิจฉัย
      1. adjustment disorder                 ตัวเอง ตนเองไมอยากฆาตัวตาย รูวา        
      2. OCD                                 ความคิดไมเขาทา       แตไมสามารถยับยั้ง
      3. PTSD                                ความคิดตนเองได จงใหการวินิจฉัย
      4. generalized anxiety disorder             1. Schizophrenia
76. จงบอกแนวทางการรักษาผูปวยที่มี               2. OCD
ความคิดฆาตัวตายที่มีความเสี่ยงสูงมาก             3. Depression
      1. นัดตรวจวันรุงขึ้น    ยาตานเศรา
                                                  4. GAD
              จิตบําบัด
                                             80. วัยรุนตองการที่จะฆาตัวตาย กาวราว
      2. นัดตรวจวันรุงขึ้น       ยาคลาย
         กังวล รักษาดวยไฟฟา                ตั้งแตอกหักจากแฟน Dx
      3. รับไวในโรงพยาบาล         ยาตาน         1. Antisocial
         เศรา จิตบําบัด                          2. Depression in adolescent
      4. รับไวในโรงพยาบาล        ยาคลาย          3. Adolescent turmoil
         กังวล จิตบําบัด                     81. เด็กอายุ 18 ป มีประวัติไมยอมไป
      5. รับไวในโรงพยาบาล         ยาตาน    โรงเรียน 2 เดือน มีพฤติกรรมกาวราวใสคน
         เศรา รักษาดวยไฟฟา
                                             ในครอบครัว ทํารายรางกายตัวเองกรีดแขน
77. บทบาทของแพทยในการดูแลผูปวยติด
                                             เปนรอยทั้งแขน          ปนขึ้นหลังคาขูวาจะ
                                                                                    
ยาเสพยติดตามพรบ.ฟนฟูสมรรถภาพผูติด
ยาเสพยติด จะเริ่มที่ขั้นตอนใด               กระโดดตึกฆาตัวตาย พี่สาวมีประวัติรักษา
      1. วินิจฉัย                            กับจิตแพทย จงวินิจฉัย
      2. คนหา                                    1. school refusal
      3. หยุดยา                                   2. adjustment disorder in
      4. ฟนฟู                                       adolescent
78. ขอใดผิดเกี่ยวกับ smoking ในผูใหญ
                                                  3. depression in adolescent
และเด็ก
                                                  4. adolescent turmoil
      1. ผูใหญตองการนิโคตินในการเลิก
                                                  5. schizophrenia
         นอยกวาเด็ก
      2. เด็กตองการมี morning nicotine      82. ขอใดจัดวาเปน psychosis
         มากกวา                                  1. ขามถนนกลัวรถชน
2. กลัวคนอื่นมาทํามิดีมิรายจากอาชีพ   85. ชายอายุ 30 ปอาชีพ graphic designer
         พนักงานตรวจตั๋วโรงหนัง             มาดวยอาการปวดทองนอยเปนๆหายๆ 2
     3. กาวราว                            ป ถายลักษณะเหลว ไมมีมูกเลือด รูสึกวา
     4. ไดยินเหมือนมีคนเรียกแตไมแนใจ    ถายไมสุด ตรวจรางกายปกติ จงวินิจฉัย
         เลยหันกลับไปดู                           1. somatization disorder
83. ผูปวยหญิง 33 ป มาหองฉุกเฉินดวย           2. somatoform pain disorder
อาการปวดหัวมาก มีประวัติ admit ดวย               3. somatoform autonomic
psychosis 3 ครั้งในชวง 3 ป ประวัติ                   dysfunction
เพิ่มเติมชวง 3 เดือนมานี้ ผูปวยชอบทํา          4. conversion disorder
อะไรซ้ําๆ เนนรายละเอียด แตงตัวแปลกไป            5. somatoform disorder unspecified
ไมสนใจการเขาสังคม แยกตัว ไมมีเพื่อน      86. ผูปวยหญิงพยายามฆาตัวตาย มีอาการ
สนิท พูดถึงแตพอแม จัดวามี personality   ของ depression เนื่องจากโดนแฟนทิ้ง
แบบใด                                       ผูปวยเติบโตมาในครอบครัวที่แตกแยก
     1. paranoid                            กอนหนานี้มีประวัติเคยพยายามฆาตัวตาย
     2. schizoid                            มาแลวหลายครั้ง ขอใดตอไปนี้ไมใชปจจัย
                                                                                  
     3. borderline                          เสี่ยงที่สําคัญของการฆาตัวตายในผูปวย
                                                                                
     4. ntisocial                           รายนี้
     5. schizotypal                               1. ครอบครัวแตกแยก
84. ชายวัย 45 ปมีความกังวลวาภรรยามีชู          2. depression
นอกใจมา 1 ป ที่ผานมาไมเคยมีหลักฐาน             3. ขาด psychological support
หรือเหตุการณใดๆที่บอกวาภรรยาจะ                  4. เคยซ้ําชั้นตอนเรียนมัธยม
นอกใจ ไมวาญาติๆและภรรยาจะอธิบาย
                                            FORENSIC
อยางไรก็ไมยอมเชื่อ เคยหึงหวงจนถึงขั้นลง   87.       ขอใดตอไปนี้ไมเปนความผิดตาม
มือทํารายภรรยา        ดานการทํางานยังไป   กฎหมายอาญา
ทํางานไดตามปกติ เปนหัวหนาแผนกใน              1. การกระทําโดยจําเปนตามมาตรา
บริษัทแหงหนึง จงใหการวินิจฉัย
              ่                                    67
     1. generalized anxiety disorder            2. การพยายามทําแทงโดยหญิงนั้น
     2. delusional disorder                        ยินยอม
                                                3. การกระทําโดยประมาทเลินเลอไม
     3. schizophrenia
                                                   รายแรง
     4. paranoid personality disorder           4. การกระทําโดยบันดาลโทสะ
88. ขอใดตอไปนี้ไมจัดเปนความผิดตาม             2.   นาหมอ ตัวทาน
กฎหมายอาญามาตรา 276 และ 277                       3.   นาหมอ เพื่อน
      1. นายดําซื้อบริการทางเพศจาก                4.   นาหมอ ตัวทาน เพื่อน
          เด็กชายอายุ 13 ป
                                                  5.   ไมมีใครผิด เพราะทําเชนนี้กันทุก
      2. นางสาวก. อายุ 14 ปมีเพศสัมพันธ
                                                       คน
          กับแฟนหนุมอายุ 18 ปอยาง
          ยินยอมพรอมใจ                       91. ปจจัยตอไปนี้มีผลทําใหเกิด rigor
      3. นายแดงอายุ 20 ปมีเพศสัมพันธ        mortis เร็วขึ้น ยกเวนขอใด
          กับแฟนสาวอายุ 20 ป แลวแอบ              1. มีไขสูง เสียชีวิตดวย septic shock
          ถายคลิปไปเผยแพรโดยแฟนสาว               2. จมน้ําและกระเสือกกระสนกอนตา
          ไมรู                                   3. CO poisoning
89. ผูปวยหญิงอายุ 16 ป มาพบแพทยดวย
                                                   4. ชักกอนตาย
เรื่องโดนลุงขมขืนกระทําชําเรา         ตรวจ
รางกายพบรอยช้ําบริเวณขาหนีบ มีแผลฉีก              5. เสียชีวตในที่อุณหภูมิสูงกวาปกติ
                                                              ิ
ขาดบริเวณอวัยวะเพศ ตองเย็บแผล ผล             92. หากทานลงความเห็นเกี่ยวกับลักษณะ
ตรวจ sperm และ acid phosphatase               ของบาดแผลโดยสุจริต แตลงความเห็น
negative ขอใดถูกเกี่ยวกับการใหความเห็น      เกี่ยวกับระยะเวลารักษาหายไมสุจริต ทาน
      1. ตรวจพบหลักฐานวา ผูปวยนาจะ
                                            จะมีความผิดตามกฎหมายใดบาง
          ถูกขมขืนกระทําชําเรา                    1. กฎหมายอาญา
      2. ตรวจพบหลักฐานวา ผูปวยนาจะ
                                                 2. กฎหมายแพง
          ผานการรวมประเวณี                       3. กฎหมายอาญา พรบ.วิชาชีพเวช
      3. ตรวจพบหลักฐานวา ผูปวยอาจจะ
                                                      กรรม
          ผานการรวมประเวณี                       4. กฎหมายอาญา พรบ.วิชาชีพเวช
      4. ตรวจไมพบหลักฐานวา ผูปวยผาน               กรรม ขอบังคับแพทยสภา
          การรวมประเวณี                      93. ผูปวยประสบอุบติเหตุ มามแตกตองตัด
                                                                    ั
90. นาของทานเปนแพทยประจํารพ.แหง          มามทิ้ง หลังผาตัดนอนโรงพยาบาลอยู 10
หนึ่ง ทานไปหานาที่รพ. เพื่อขอใบรับรอง       วันไมมีภาวะแทรกซอน สามารถกลับบาน
แพทยไปสมัครเรียนวายน้ํา นาของทาน          ได ทานจะลงความเห็นในใบรับรองแพทย
เขียนใบรับรองแพทยใหตวทาน และเขียน
                           ั                  วาอยางไร
อีกหลายใบเผือใหเพื่อนๆที่จะสมัครเรียน
                 ่                                 1. พักรักษาตัวเปนเวลา 10 วัน หายดี
ดวยกัน กรณีเชนนี้ ผูใดมีความผิดบาง             2. ไดรับบาดเจ็บสาหัสเนื่องจาก
      1. นาหมอ                                        สูญเสียอวัยวะ
3. อาจเปนอันตรายถึงแกชีวตไดหาก
                                ิ           tonsillar exudate both sides, bulging of
        ไมไดรับการรักษาอยางทันทวงที     lateral pharyngeal wall ควรใหการรักษา
    4. ไดรับบาดเจ็บตองทนทุกขเวทนา         อยางไร
                                                 1. aspiration and oral ATB
        เปนเวลามากกวา 21 วัน จัดวา
                                                 2. oral ATB only
        บาดเจ็บสาหัส                             3. IV ATB only
94. โจทยใหรูปหลังของผูปวย มี pattern         4. external drainage and ATB
abrasion เปนรูป tram line 5 รอยที่กลาง          5. tonsillectomy
หลัง ขอใดผิด                               98. ผูปวยอายุ 40 ปมีอาการบานหมุน
    1. ถูกตีดวยอาวุธเหลี่ยม มีสันอยาง     ขึ้นมาทันทีรวมกับเวียนศีรษะคลื่นไส
        นอย 5 ที                           อาเจียนอยางมาก ไมมีเสียงในหู การไดยิน
                                            ปกติ อาการบานหมุนไมสัมพันธกับทาทาง
    2. ถูกตีดวยอาวุธลักษณะกลมอยาง
                                            หรือการเคลื่อนไหวของศีรษะ จงใหการ
        นอย 5 ที                           วินิจฉัย
    3. ถูกตีดวยอาวุธกลวงอยางนอย 5 ที          1. Meniere disease
    4. ถูกตีดวยอาวุธกลมหรือเหลี่ยมก็ได         2. positional vertigo
        แตไมสามารถบอกจํานวนครั้งได            3. vestibular neuritis
                                                 4. paroxysmal          nonpositioning
ENT                                                  vertigo
95. ด็กชาย 4 ขวบ มาดวย greenish            99. เด็กหญิงอายุ 10 ปเปนนักวายน้ําของ
discharge from Lt. nostril กินยา ATB มา     โรงเรียน มาโรงพยาบาลดวยมี greenish
หลายขนานแลวไมหาย ควรทําอะไร               nasal discharge และมีอาการปวดที่โหนก
    1. ตรวจหา foreign body and              แกมและฟนกรามบนดานขวา ตรวจรางกาย
        remove                              พบ greenish nasal discharge สอง
    2. culture for fungus and treat         nasopharynx พบวามี discharge สีเขียว
    3. culture and ATB                      เชนกันและมี mild tenderness over
    4. CT sinus                             maxillary sinus area จงใหการรักษา
96. Organic solvent ที่มีหลักฐานวาทําให        1. topical decongestant only
เกิด hearing loss คือสารใด                       2. topical decongestant + oral ATB
    1. Toluene                                   3. topical decongestant + oral
    2. Styrine                                       antihistamine
    3. Xylene                                    4. topical decongestant + nasal
97. ปวยชาย 25 ปมาดวยไขสง คอบวม อา
                             ู                       steroid
ปากไมขึ้น ตรวจรางกายพบวา มี trismus,
100. เด็กชาย 10 ป เปนหูน้ําหนวกมา              4. IV dobutamine 0.35 mg/min
ตั้งแต 8 ป มี discharge กลิ่นเหม็นออกมา        5. IV isoproterenol 5.0 mcg/min
เกือบทุกวัน ตรวจ otoscope พบวามี attic     103. A thai male, U/D alcoholism
perforation with keratin debris จงใหการ
                                            presented with orthopnea, PND, JVP 3
รักษา
     1. ATB eardrop                         cm, S3 gallop, cardiomegaly, pretibial
     2. myringoplasty with tube             pitting edema, warm extremities. Which
     3. modified radical mastoidectomy      is the most appropriate management?
     4. radical mastoidectomy                    1. Lasix 80 mg IV bolus
101. ผูปวยหญิง ประวัติเปน allergic            2. Lasix 300 mg IV drip in 24 hour
rhinitis 3 วันกอนมี purulent discharge,         3. Thiamine 100 mg IV
nasal congestion, PE: tender both
                                                 4. Dobutamine
frontal sinus จะสง investigation ใด
     1. nasal decongestant                  104. A pregnant woman is diagnosed
     2. nasal decongestant + amoxicillin    with P. falciparum malaria. What is the
     3. nasal decongestant +                appropriate medication?
         doxycycline                             1. IV quinine
     4. nasal decongestant + CT                  2. IV artesunate
     5. nasal decongestant +
                                                 3. Artesunate+mefloquine
         radiograph
                                                 4. Artesunate+doxycycline
MED
102. A 48 years old male was admitted            5. Chroloquine+primaquine
to coronary care unit with an acute         105. A man has had exertional angina
inferior wall myocardial infarction. Two    and syncope. His blood chemistry
hours after admission, his BP = 86/52       shows total cholesterol 250 and PE
HR 40 bpm with sinus rhythm. Which of       reveals DBP> 90 on several occasion.
the following would be the most
                                            What is the diagnosis
appropriate initial therapy?
                                                 1. Mitral stenosis
     1. Immediate insertion of a
                                                 2. Mitral regurgitation
         temporary transvenous
                                                 3. Aortic stenosis
         pacemaker
                                                 4. Aortic regurgitation
     2. IV atropine sulphate 0.6 mg
                                                 5. Tricuspid stenosis
     3. IV NSS 300 mL in 15 minutes
106. 70 yr-old woman with underlying           2. bilateral carotid sinus massage
disease of DM has watery diarrhea. She         3. IV adenosine
said that 5 days ago she had dysuria           4. IV lidocaine
and diagnosed to have UTI.                     5. synchronized cardioversion
Ciprofloxacin was prescribed then.         109. What is the advantage of using
Which of the followings has the LEAST      colloid solution compared with
benefit?                                   crystalloid?
    1. sigmoidoscope                           1. Colloid solution has bigger
    2. stool for C. difficile toxin                 molecule and doesn’t cross
    3. Ba enema                                     plasma membrane.
    4. stool occult blood                      2. การให colloid ใชอัตราสวน 1:1
    5. metronidazole 400 mg bid 7 days              เทียบกับเลือด
107. A 50 yr-old woman had chest pain          3. 40 cc of fluid was held in the
3 hrs ago. At ER, her EKG shows                     plasma for every 1 g of colloid
inverted T wave in lead II, III and aVF.            infused.
Her initial medication includes O2             4. No evidence stated that colloid
therapy, IV NTG, beta-blocker and ASA.              is better than crystalloid.
What is the next proper management?            5. colloid solution is not at risk for
    1. IV thrombolytic agent                        transmitted disease
    2. Percutaneous angioplasty            110. A 51 yr-old woman with underlying
    3. IV heparin                          COPD and prolonged intubation for
    4. emergency CABG                      hypercarbic respiratory failure. Her
108. A man came to ER because of           current UA shows WBC 3-5/hpf, many
palpitation and dizziness. EKG reveals     yeast. She is afebrile, clinically stable.
supraventricular tachycardia with          Foley catheter draining dark yellow
ventricular rate of 150 bpm. Right         urine. Urine culture grow >10000
carotid sinus massage was done but no      colonies of C. albicans. What’s the most
response was observed. What is the         appropriate management?
next management?                               1. treat only if U/C grow >100000
    1. continue carotid sinus massage               colonies
2. treat only if repeat UA show >25   followings is NOT appropriate
       WBC/hpf                            management?
    3. give fluconazole via NG                 1. Insulin IV push
    4. off Foley catheter                      2. NSS IV load
    5. intravesicular amphotericin B           3. IV KCl
111. A 55 yr-old man has substernal            4. IV bicarbonate
chest pain for 2 hrs. The pain was dull        5. empirical ATB
and radiated to Lt. shoulder but not to   113. A patient sustained spinal cord
the arm. At ER, 12-lead EKG was done.     injury from T6 level. He came to ER
Which of the following management         with tachycardia, plethora and
should NOT be done?                       diaphoresis. He claimed that he missed
                                          his routine urinary catheterization today.
                                          What is the appropriate management?
                                               1. bed rest with leg elevation
                                               2. urinary catheterization
                                               3. stellate ganglion block
                                          114. A 45 yr-old healthy woman has
                                          headache for 2 weeks. PE: reveals stiff
                                          neck positive. LP was done. CSF open
    1. gastric lavage                     pressure 28 cmH2O, WBC 250 (L>N)
    2. IV PPI                             protein 400, sugar 25 (plasma glucose
    3. upper GI endoscopy                 100), no bacteria in gram stain. What is
    4. Oral Antacid                       your diagnosis?
    5. nutritional support                     1. TB meningitis
112. A 75 yr-old female was brought to         2. Aseptic meningitis
the hospital because of alteration of          3. Purulent meningitis
consciousness. PE reveals fever,               4. Eosinophilic meningitis
lethargy, tachypnea. ABG: pH=7.26              5. Cryptococcal meningitis
PCO2=56 PaO2=35 HCO3=26. DTx
shows hyperglycemia. Which of the
115. Middle aged man came to ER             CBC: Hb 7, MCV 102, RDW 20%,
because of sudden severe headache,          normal WBC and plt. What test should
sudden onset of Rt. hemiparesis and         be done for the definitive diagnosis?
numbness. He was alert at first                  1. HAM test
presentation but later he lost his               2. Coombs’ test
consciousness. PE: right pupil fixed             3. Stool occult blood
dilated. After that, he has respiratory          4. G-6-PD assay
arrest. What is the most likely                  5. Hb typing
diagnosis?                                  118. A patient with underlying disease
     1. Superior      sagittal      sinus   of chronic hepatitis C infection had
        thrombosis                          percutaneous liver biopsy 3 days ago.
     2. Cerebellar hemorrhage               Today he develops fever, RUQ pain,
     3. Pontine hemorrhage                  jaundice and melena. What is the most
     4. Diffuse subarachnoid                likely diagnosis?
        hemorrhage                               1. Hemobilia
     5. Medullary hemorrhage                     2. Hepatitis from reactivation
116. A man was bitten by a snake with       119. A 20 yr-old woman came to you for
black and white stripe in Rayong            routine check up. She weighs 97 kg
province. At ER, PE reveals fang            with 153 cm in height. V/S WNL. PE:
marks. Later he developed ptosis,           unremarkable. AST 34 ALT 123 HBs Ag
dyspnea and muscle weakness. Which          and anti-HCV negative. What is the
of the following snake has bitten him?      diagnosis?
     1. cobra                                    1. HAV infection
     2. king cobra                               2. HBV infection
     3. Malayan krait                            3. HCV infection
     4. Malayan pit-viper                        4. Fatty liver
     5. Russel viper                        120. A24 yr-old adolescent had sudden
117. 40 yr-old woman developed fatigue      onset of pleuritic chest pain and
and dyspnea for 3 wks. PE: moderately       dyspnea for 1 hour. He is a tall, thin
pale, mild jaundice, mild splenomegaly.     man with no underlying disease. PE:
decrease Rt. breath sound, RR 24/min,      this event. Today he presents with
normal BP and PR. Chest X-ray shows        painful and swollen joint. PE reveals
Rt. pneumothorax 50%, otherwise WNL.       painful and swollen Rt. knee joint with
What is the appropriate management?        limited ROM due to pain. His
     1. Tube thoracostomy                  coagulogram shows prolonged APTT,
     2. IV fluid                           normal PT. The laboratory abnormality
     3. Consult CVT for video assisted     can be corrected by mixing study. And
         thoracotomy                       when the patient’s serum is mixed with
     4. Observe                            the serum from hemophilia A patient, it
121. A 65 yr-old man was admitted          shows prolonged APTT. What is the
because he had post CVA left               most appropriate management?
hemiparesis. 5 days later, he develops          1. DDAVP
Lt. leg edema. PE: left thigh edema, not        2. Cryoprecipitate
tender. What is the investigation for           3. Cryo-removed plasma
definitive diagnosis?                           4. FFP
     1. D-dimer                                 5. PRC transfusion
     2. lymphangiography                   124. A 60 yr-old man with heavy
     3. contrast radiograph                smoking, presented with progressive
     4. radionucleotide venography         dyspnea and dyspnea on exertion. His
     5. Doppler USG                        symptom now is stable. He has
122. Which is the least likely to happen   yellowish phlegm 5-6 cc every day. PE:
in lightning injury?                       bilateral rhonchi Hct 49% Which of the
     1. apnea                              following treatment has been showed to
     2. cardiac standstill                 prolong his survival?
     3. immediate muscle necrosis
                                              1.   ACEI
     4. keraunoparalysis                      2.   theophylline
     5. ruptured tympanic membrane            3.   Oxygen
123. A 20 yr-old man has a history of         4.   Steroid
bleeding diathesis since he was a child.      5.   beta-agonist
No one in the family has experienced
125. A 39 yrs-old male visits your office       4. need surgical correction in bed-
for routine check up. His lipid profile             ridden case
shows cholesterol 150 mg/dl and                 5. cardiomyopathy is a clinical
triglyceride 300 mg/dl. His father died             feature
from acute MI 4 yrs ago. What is the        128. 40 yr-old female came to the
most appropriate initial management?        hospital because she had fever and
     1. Dietary modification                acute RUQ pain. PE: BT 38°C,
     2. Exercise                            tenderness at RUQ, no rebound
     3. Gemfibrozil                         tenderness. What is the most
     4. Cholestyramine                      appropriate investigation?
     5. clofibrate                              1. Plain film abdomen
126. A nurse had a history of contacting        2. USG upper abdomen
person known to have pulmonary TB.              3. UGI study
She came to visit you at your office.           4. CT
From her medical record, PPD skin test          5. MRI
was done last year with a result of 0.3     129. Which is INCORRECT about DVT?
cm at 48 hr. You send her to have PPD           1. Virchow triad consists of
skin test again which shows an                      hypercoagulable state,
induration of 13 mm at 48 hr. What                  endothelial injury and pulmonary
should you do next?                                 embolism
     1. CXR                                 130. ชาย 25 ปคัดจมูกตลอดทั้งป ตรวจพบ
     2. INH 300 mg/day for 3 months         postnasal drip, granular pharynx ไมมี
     3. check baseline liver enzymes        ประวัติครอบครัวเรื่องภูมิแพ ไมมีประวัติ
     4. check liver enzyme every 3          asthma, eczema ตรวจพบ eosinophil
         months                             มากใน nasal discharge การตรวจอะไรจะ
     5. repeat PPD skin test again next     ชวยในการวินจฉัยมากที่สุด
                                                         ิ
         week                                   1. serum IgE level (radiodiffusion)
127. Which is INCORRECT about                   2. serum IgE level (radio
muscular dystrophy?                                 immunoabsorbent assay)
     1. probably muscle enzyme defect           3. food elimination test
     2. muscle wasting in specific              4. skin test
         muscle group
     3. genetically determined
131. ผูปวยหญิง 30 ป เปน Graves’            1. penicillin
disease กินยา PTU อยูที่ 150 mg/day มี        2. dicloxacillin
maculopapular rash ขึ้นตามตัว ควรทํา           3. ceftriaxone
                                               4. ciprofloxacin
อยางไร
                                               5. gentamicin
    1. เพิ่ม dose เปน 300 mg/day          135. ผูปวยหญิงอายุ 75 ปญาติพามา
    2. ลด dose ลงเปน 100 mg/day           โรงพยาบาลดวยอาการซึม              ไมคอย
    3. เปลี่ยนเปน MMI 7.5 mg/day          ตอบสนอง ตรวจรางกาย BP 110/90 PR
    4. หยุดยาแลวเปลี่ยนเปน     lithium   54/min RR 10/min ซีด ไมเหลือง ฟงไดยิน
        carbonate
                                           เสียงหัวใจเบาลง ไมมี murmur เสียง
    5. หยุดยาแลวเปลี่ยนเปน
                                           bowel sound ลดลง ไมมี melena เจาะ
        prednisolone
                                           เลือดไดผลดังนี้
132. สารใดตอไปนี้สัมพันธกับภาวะ          CBC: Hb 10.1 WBC 5100 plt. 130000
hyperventilation นอยที่สุด                DTx 67 mg/dl
    1. methamphetamine                     E’lyte: Na 126 Cl 109 K 4.5 HCO3 27
    2. cocaine                             ABG: pH 7.23 pCO2 64 pO2 72 HCO3
    3. alcohol
                                           27
    4. marijuana
133. ผูปวยไตวายเรื้อรัง มาพบแพทย        ทานจะสง investigation อะไรเพิ่มเติมเพื่อ
                                           การวินิจฉัย
แพทยแจงวาจะตองลางไตในอีก 1-2 ป
                                               1. serum cortisol, insulin level, C-
ขางหนา ผูปวยไมเคยฉีดวัคซีนใดๆเลย
                                                    peptide
ทานจะแนะนําใหผูปวยฉีดวัคซีนใด ยกเวน
                                               2. TSH, free thyroxine
    1. annual influenza virus
                                               3. serum Ca, PTH
    2. HAV vaccine
                                               4. EKG, echocardiogram
    3. HBV vaccine
                                               5. urine toxicology screen
    4. polyvalent           pneumococcal
                                           136. ขอใดไมใช hypersensitivity
        peptide vaccine
                                               1. MP rash + liver enzyme
    5. varicella vaccine
                                                    increase หลังกิน navirapine 2 วัน
134. ผูปวยเบาหวาน ควบคุมระดับน้ําตาล         2. facial angioedema หลังกิน ACEI
ไดดี เปน perianal abscess ควรใหยาตัว             3 วัน
ใด
3. wheal and flare หลังกิน penicillin    characteristics ปกติดี PR มี mass
      2 ชั่วโมง                             anterior to rectum จงใหการวินิจฉัย
   4. มี target lesion หลังกิน                   1. Vaginal agenesis
                                                 2. Imperforated hymen
      sulfonamide 5 วัน
                                                 3. Bicornuate uterus
ANES                                             4. Gonadal agenesis
137. Spinal anesthesia should not be        141. ทา LSA หมายถึงทารกอยูทาใด
done in                                          1. Transverse
    1. patient with high fever                   2. Cephalic presentation
    2. patient with history of post dural        3. Breech presentation
         puncture headache from                  4. blow presentation
         previous SAB
                                                 5. Face presentation
    3. patient with mild scoliosis
    4. patient with asthma                  142. ผูหญิง GA 38 weeks มาดวยอาการ
    5. severe pre-eclampsia patient         เจ็บครรภ ตรวจพบ deceleration not
138. ยาใดไมควรใหเปน induction agent      associated with uterine contraction
ในรายที่มี increased ICP                    ทารกอยูในภาวะใด
    1. Thiopental                                1. Utero-placental insufficiency
    2. Ketamine                                  2. Vagal stimulation
    3. Propofol
                                                 3. Oligohydramnios
    4. midazolam
    5. etomidate                            143. หญิงตั้งครรภอายุ 30 ป GA 16
139. A 70 yr-old man has pain after an      สัปดาห มาพบแพทย และขอใหแพทยทา      ํ
operation. What is the appropriate post-    การเจาะน้ําคร่ําไปตรวจ chromosome
op pain management?                         เนื่องจากรับไมไดถาลูกจะเปน down
    1. paracetamol 500 mg prn               syndrome ถาทานเปนแพทยคนนั้น ทาน
    2. pethidine 25 mg IM q..h
                                            จะทําอยางไร
    3. MO 10 mg IM q 6 h
                                                 1. ทําตามที่ผูปวยตองการ
                                                                 
    4. MO 5 mg IV q 3 h
OB-GYN                                           2. บอกวาความเสี่ยงจากการเจาะมีสูง
140. ผูปวยหญิง 15 ป มาดวยปวดทองทุก              กวาความเสี่ยงที่ลูกจะเปน down
เดือน (30 วัน) ครั้งหนึงเปน 1-2 วันก็
                         ่                           syndrome จึงไมควรเจาะ
หายไป ไมเคยมีประจําเดือนมากอน ตรวจ             3. บอกวาผูปวยกังวลมากเกินไป ไม
                                                               
รางกายพบวามี secondary sexual                     ตองเปนหวง
4. บอกวาอายุขนาดผูปวย โอกาสที่       147. What is the most likely to be found
        ลูกจะเปน down syndrome มีไม       in infant born from mother who drank a
        เยอะ ใหสบายใจได                   lot in first trimester?
    5. ใหไปพบแพทยคนอื่น                        1. Cleft lip, cleft palate
144. Most common cause of uterine                2. Microcephaly
rupture between pregnancy is?                    3. hydrocephalus
    1. การแยกของแผลผาตัด C/S ครั้ง         148. Which of the following ATB is
        กอน                                contraindicated in pregnancy?
    2. Prolonged labor                           1. Ceftriaxone
    3. Unattended labor with oxytocin            2. augmentin
        infusion                                 3. metronidazole
    4. Fetal malposition                         4. azithromycin
145. Pregnant woman HIV positive,                5. ofloxacin
CD435 ตรวจรางกายพบ Oral candidiasis        149. ผูหญิงทอง GA 32 wk. ปวดทองมา
มีอาการไอเรื้อรัง CXR: cavitation AFB 3+    12 ชั่วโมง เริ่มจาก periumbilical ตอมา
which is the least appropriate              ยายมาปวดที่ RLQ, PE: guarding at RLQ
management                                  จงใหการรักษา
    1. ให IRZE                                  1. Ultrasound
    2. Start AZT, 3TC, efavarenz                 2. Appendectomy
    3. Nevirapine ไมควรใชในผูปวยราย
                                                3. Observe
        นี้                                      4. ATB
    4. Plan ที่จะ start ARV โดยเร็วที่สุด        5. Emergency exploratory
        หลังเริ่ม Anti TB drug                        laparotomy
146. What is the most common                150. ในผูปวยรายที่มี asymmetrical IUGR
symptom of HEELP syndrome?                  และตรวจพบเปน uteroplacental
    1. epigastrium pain                     insufficiency ทํา Doppler ultrasound จะ
    2. headache                             พบอะไร
    3. pulmonary edema                           1. Increase umbilical artery
    4. blurred vision                                 resistance
2. Decrease umbilical artery            154. หญิงอายุ 55 ป มี abdominal
      resistance                           discomfort PE: abdomen mild distension,
   3. Increase umbilical artery            ascites. USG: suspected left ovarian
                                           mass จงบอก chance ที่จะเปน early
      systemic flow
                                           ovarian CA
   4. Increase MCA systolic flow
                                               1. 5%
   5. Increase MCA resistance
                                               2. 10%
151.GDM class C หลังคลอดตองลด                 3. 25%
insulin ที่ใหลงเนื่องจากอะไร                  4. 50%
    1. decrease in human chorionic             5. 75%
        somatomammotropin
                                           EYE
152. ขอใดผิด เกี่ยวกับการขอ consent ทํา   155. ผูปวยเด็ก 5 ขวบ แมพามาพบจักษุ
laparoscopic tubal ligation                แพทยดวยเรืองตาขางขวาเหลเขาในตั้งแต
                                                         ่
                                           อายุ 5 เดือน
   1. ตอง advice วาจะไมสามารถมีบุตร     ตรวจตา VA: RE 20/100 with PH 20/70
       ไดอีก                                              LE: 20/30
   2. ตองบอกสามีของผูปวยวาภรรยาจะ               esotropia         45 prism
       ไมสามารถมีบุตรได                           myopia RE +0.5 LE +0.25
   3. ตอง advice วาไมรับรองผล 100%               no limit of motion
                                           ตรวจรางกายอื่นๆอยูในเกณฑปกติ ผูปวย
   4. ถาปวดทอง หรือ ไมมีประจําเดือน
                                           ไดรับการวินิจฉัยวาเปน esotropia with
       หลังทํา ตองรีบมาโรงพยาบาล          amblyopia RE Management?
   5. แนะนําวิธีอื่นในการคุมกําเนิด เชน        1. observe อาจหายไดเองตอนโตขึ้น
       vasectomy, condom                            นัด F/U ที่ 1 ป
153. Known case CA ovary มาดวยซึม              2. ใสแวนสายตาตามที่วัดได
ตรวจพบ Na 135 K 4 Cl 103 HCO3- 23               3. CT scan เพราะอาจมีความ
                                                    ผิดปกติของสมองได
Ca 18 จงให initial management
                                                4. ปดตาขางที่ดีเพื่อรักษา amblyopia
   1. IV NSS keep urine output 200-
                                                5. ผาตัดกลามเนื้อลูกตา
       300 mL/hr                           156. ผูปวยเด็ก มารดาพามาตรวจดวยเรือง ่
   2. Calcitonin injection                 ตาเหล วินิจฉัยวาเปน pseudoesotropia
   3. bisphosphonate                       เนื่องจากอะไร
   4. Hemodialysis                              1. telecanthus
2. epicanthus กวาง                   protruding at groin region but it can’t be
     3. narrow palpebral fissure           reduced anymore. He has no fever. The
SURG                                       mass is tender. What is the LEAST
157. A 60 yr-old man has reducible right
                                           appropriate management?
groin mass for 2 years. But 12 hrs ago
                                               1. Try reduction is safe.
he developed pain at groin mass. He
tried to reduce the mass but found that        2. He needs emergency surgery
it is irreducible. He also had nausea              because this is an emergency
and vomiting. PE: HR 100 BT 38oC. Rt.              condition.
groin mass 5 cm, irreducible with              3. Elective herniorrhapy can be
tenderness. What is the appropriate                scheduled.
management?
                                           160. Which is correct about papillary
     1. Analgesic and muscle relaxant +
                                           carcinoma of bladder?
         trying reduction
                                               1. mostly asymptomatic
     2. Analgesic and muscle relaxant +
                                               2. rarely metastasize
         observe
                                               3. could be treated by TUR-BT
     3. Emergency herniorrhaphy
                                           ETHICS
     4. Emergency laparotomy
158. ผูปวยชาย 70 ป มีอาการ lower        161. ผูปวยเปน end stage lung CA
urinary tract symptoms (LUTS) PE:          แพทยไดปรึกษาญาติถง prognosis ผูปวย
                                                                  ึ
prostate hypertrophy with firm nodular     และญาติตัดสินใจกันวา จะไมใสทอชวย
                                           หายใจ หลังจากนั้น 2 วัน ผูปวยเกิด  
at right lobe. What is the most
                                           cardiopulmonary arrest ญาติขอรองให
appropriate investigation?
                                           แพทยชวยกูชวิตเพื่อรอบุตรสาวกลับมาจาก
                                                         ี
     1. ultrasound KUB                     ตางจังหวัด แพทยจึงได CPR และใสทอ
     2. transrectal ultrasound with        ชวยหายใจ วันรุงขึ้นญาติเกิดเปลี่ยนใจ จึง
         biopsy                            อยากใหแพทยชวยถอดเครืองชวยหายใจ
                                                                     ่
     3. PSA                                ออก ขอใดถูกตอง
     4. CT scan                                1. ไมสามารถทําได เพราะถือเปนการ
159. 50 yr-old man had history of                  คนตายโดยเจตนา
reducible groin mass for 8 years. 2            2. ทําได เพราะผูปวยเคยหยุดหายใจ
hours PTA, he noticed a mass                       ไปครั้งหนึ่งแลว และเปนการทํา
                                                   ตามที่ผูปวยไดแสดงความจํานงไว
                                                             
3. ทําได แตใหญาติเซ็นอนุญาตกอน      164. ผูปวยชาย อายุ 25 ป ใสคอนแทค
      4. ทําได เพราะเคยตกลงกับญาติไว        เลนสนอน ตื่นเชามาเจ็บตา ไปหาหมอ
          กอนหนานี้แลว                     ปรากฎวาเปน corneal abrasion ควรทํา
      5. ทําได       แตใหญาติเปนฝายถอด   อยางไร
          เครื่องชวยหายใจ                        1. หยอดยาชา กลับบาน
162. จากคําประกาศสิทธิผูปวย บิดา                2. topical steroid
มารดา หรือผูแทนโดยชอบธรรม อาจใช                 3. topical ATB + pressure patch
สิทธิแทนผูปวยที่เปนเด็กอายุยังไมเกิน ...
             
                                              165. ผูปวยหมดสติ EKG เปน Pulseless
ป ผูบกพรองทางกายหรือจิต ซึ่งไมสามารถ      Electrical activity ควรทําอะไร
ใชสิทธิดวยตนเองได                              1. Defibrillation
      1. 7 ป                                     2. Atropine
      2. 15 ป                                    3. Adenosine
      3. 17 ป                                    4. Lidocaine
      4. 18 ป                                166. การศึกษาหนึ่ง แบงกลุมผูเขารวม
      5. 20 ป                                การศึกษาเปน2 กลุมไดแก ผูปวยมะเร็งรัง
ADD ON                                        ไข และคนปกติ แลวถามถึงประวัติการกิน
163. เด็กอายุ 6 ป ไขสูง ซึมลงและมีอาการ     ยาคุมกําเนิด ถามวาเปนการศึกษาแบบใด
ชัก อาศัยอยูแถบชานเมือง ตรวจรางกาย              1. Experimental study
ไมรูสึกตัว รูมานตา 4 มิลลิเมตร ตอบสนอง
                                                 2. cohort study
ตอแสงเล็กนอย คอไมแข็ง รีเฟล็กซ                3. historical cohort study
คอนขางไว ผลตรวจน้ําไขสันหลัง ความดัน            4. case control study
180/100 มิลลิเมตรน้ํา เซลล 350 (L 80%        167. ผูปวยมาดวยประวัติ hearing loss
N 20%) น้ําตาล 70 (น้ําตาลในเลือด 100)        ขางเดียว ตรวจหูพบ Retraction of TM Rt.
โปรตีน 50 จงใหการวินิจฉัย                    ตรวจ Tympanogram ขางขวา type B สวน
      1. Toxic encephalopathy                 ขางที่ปกติ type A ควรทําอยางไร
      2. Viral meningitis                         1. ตรวจ nasopharynx
      3. Purulent meningitis
      4. Tuberculous meningitis
      5. Fungal meningitis
168. Which of the followings is TRUE          3. Remove blood clot and chemical
concerning thyroglossal duct cyst?                cautery
    1. It was found at lateral of neck        4. Packing
    2. Origin ถึง Base of tongue          170. เด็ก 10 ป มาดวย UGIB หลังจาก
169. เด็กชายมาดวยประวัติเลือดกําเดา      resuscitation แลว ควรทําอะไรตอไป
ออกเปนๆหายๆ มา รพ. เลือดหยุดแลวจะ           1. EGD
ทําอะไรตอไป
    1. give vaseline ointment
    2. Remove blood clot and electrical
         cautery



          อะเสวนา ไฮเบญจะ ดอคจุฬา จะพาลานัง
          บัณฑิตานัง แอคขลัง และเทวัง จะเสวนา
                                                                         พี่หนุม


              AC มิใชสรณะ โปรดใชวิจารญาณในการอาน


                                                              Topo MDCU 60
                                                                    5/3/2552
                                                                       23.45

More Related Content

What's hot

Step3 Tutorial by SWU book1
Step3 Tutorial by SWU book1Step3 Tutorial by SWU book1
Step3 Tutorial by SWU book1
vora kun
 
คู่มือเวชปฏิบัติหัตถการสำหรับ นศพ
คู่มือเวชปฏิบัติหัตถการสำหรับ นศพคู่มือเวชปฏิบัติหัตถการสำหรับ นศพ
คู่มือเวชปฏิบัติหัตถการสำหรับ นศพ
Fone Rati
 
หัตถการที่จำเป็นทางสูติ
หัตถการที่จำเป็นทางสูติหัตถการที่จำเป็นทางสูติ
หัตถการที่จำเป็นทางสูติ
vora kun
 
Osce ศรว ครั้งที่สอง 10jan53
Osce ศรว ครั้งที่สอง 10jan53Osce ศรว ครั้งที่สอง 10jan53
Osce ศรว ครั้งที่สอง 10jan53
vora kun
 
Compre si 2010 l
Compre si 2010 lCompre si 2010 l
Compre si 2010 l
vora kun
 
การซักประวัติการเจ็บป่วย
การซักประวัติการเจ็บป่วยการซักประวัติการเจ็บป่วย
การซักประวัติการเจ็บป่วย
Ozone Thanasak
 
Osce คณะ si 115
Osce คณะ si 115Osce คณะ si 115
Osce คณะ si 115
vora kun
 
Compre si 2010 ans
Compre si 2010 ansCompre si 2010 ans
Compre si 2010 ans
vora kun
 
11แผน
11แผน11แผน
11แผน
Fmz Npaz
 

What's hot (20)

For extern
For externFor extern
For extern
 
Step3 Tutorial by SWU book1
Step3 Tutorial by SWU book1Step3 Tutorial by SWU book1
Step3 Tutorial by SWU book1
 
Example osce
Example osceExample osce
Example osce
 
คู่มือเวชปฏิบัติหัตถการสำหรับ นศพ
คู่มือเวชปฏิบัติหัตถการสำหรับ นศพคู่มือเวชปฏิบัติหัตถการสำหรับ นศพ
คู่มือเวชปฏิบัติหัตถการสำหรับ นศพ
 
หัตถการที่จำเป็นทางสูติ
หัตถการที่จำเป็นทางสูติหัตถการที่จำเป็นทางสูติ
หัตถการที่จำเป็นทางสูติ
 
Osce ศรว ครั้งที่สอง 10jan53
Osce ศรว ครั้งที่สอง 10jan53Osce ศรว ครั้งที่สอง 10jan53
Osce ศรว ครั้งที่สอง 10jan53
 
Key word osce
Key word osceKey word osce
Key word osce
 
Compre si 2010 l
Compre si 2010 lCompre si 2010 l
Compre si 2010 l
 
Nle step 2_2553
Nle step 2_2553Nle step 2_2553
Nle step 2_2553
 
การซักประวัติการเจ็บป่วย
การซักประวัติการเจ็บป่วยการซักประวัติการเจ็บป่วย
การซักประวัติการเจ็บป่วย
 
Osce คณะ si 115
Osce คณะ si 115Osce คณะ si 115
Osce คณะ si 115
 
Skill manual removal of placenta
Skill manual removal of placentaSkill manual removal of placenta
Skill manual removal of placenta
 
National test _2553_TU
National test _2553_TUNational test _2553_TU
National test _2553_TU
 
Case study : dengue fever
Case study : dengue feverCase study : dengue fever
Case study : dengue fever
 
Compre si 2010 ans
Compre si 2010 ansCompre si 2010 ans
Compre si 2010 ans
 
สอบ-ศรว-มีนาคม-2551
สอบ-ศรว-มีนาคม-2551สอบ-ศรว-มีนาคม-2551
สอบ-ศรว-มีนาคม-2551
 
Case01
Case01Case01
Case01
 
แนวทางการดำเนินงานเฝ้าระวัง โรคติดเชื้อไวรัสซิกา
แนวทางการดำเนินงานเฝ้าระวัง โรคติดเชื้อไวรัสซิกาแนวทางการดำเนินงานเฝ้าระวัง โรคติดเชื้อไวรัสซิกา
แนวทางการดำเนินงานเฝ้าระวัง โรคติดเชื้อไวรัสซิกา
 
11แผน
11แผน11แผน
11แผน
 
Case teleconference 30 พย.60
Case teleconference 30 พย.60Case teleconference 30 พย.60
Case teleconference 30 พย.60
 

Viewers also liked

การอ่านค่า Ekg
การอ่านค่า Ekgการอ่านค่า Ekg
การอ่านค่า Ekg
techno UCH
 
Survivor NT step2 SIRIRAJ book 2
Survivor NT step2 SIRIRAJ book 2Survivor NT step2 SIRIRAJ book 2
Survivor NT step2 SIRIRAJ book 2
vora kun
 
ประชุมวิชาการ ศิริราช 52
ประชุมวิชาการ ศิริราช 52ประชุมวิชาการ ศิริราช 52
ประชุมวิชาการ ศิริราช 52
vora kun
 
ประชุมวิชาการ ศิริราช 53
ประชุมวิชาการ ศิริราช 53ประชุมวิชาการ ศิริราช 53
ประชุมวิชาการ ศิริราช 53
vora kun
 
Mdcu Obstetrics Tutorial
Mdcu Obstetrics TutorialMdcu Obstetrics Tutorial
Mdcu Obstetrics Tutorial
vora kun
 
Mdcu Comprehensive Cardio
Mdcu Comprehensive CardioMdcu Comprehensive Cardio
Mdcu Comprehensive Cardio
vora kun
 
Mdcu Neonatology Review
Mdcu Neonatology ReviewMdcu Neonatology Review
Mdcu Neonatology Review
vora kun
 
Total parenteral nutrition
Total parenteral nutritionTotal parenteral nutrition
Total parenteral nutrition
vora kun
 
Mdcu Preventive Screening
Mdcu Preventive ScreeningMdcu Preventive Screening
Mdcu Preventive Screening
vora kun
 
Survivor NT step2 SIRIRAJ book 1
Survivor NT step2 SIRIRAJ book 1Survivor NT step2 SIRIRAJ book 1
Survivor NT step2 SIRIRAJ book 1
vora kun
 
Abnormal pap smear ศิริราช ppt
Abnormal pap smear ศิริราช pptAbnormal pap smear ศิริราช ppt
Abnormal pap smear ศิริราช ppt
vora kun
 
Endocrine Med 2010 Step2
Endocrine Med 2010 Step2Endocrine Med 2010 Step2
Endocrine Med 2010 Step2
vora kun
 
Mdcu Step2 Gen Sx Ii
Mdcu Step2 Gen Sx IiMdcu Step2 Gen Sx Ii
Mdcu Step2 Gen Sx Ii
vora kun
 
Step3 Tutorial by SWU book2
Step3 Tutorial by SWU book2Step3 Tutorial by SWU book2
Step3 Tutorial by SWU book2
vora kun
 
Survival for all draft 1 - 1
Survival for all draft 1  - 1Survival for all draft 1  - 1
Survival for all draft 1 - 1
Domo Kwan
 

Viewers also liked (20)

ศรว 51 ANS By Cmu
ศรว 51 ANS By Cmuศรว 51 ANS By Cmu
ศรว 51 ANS By Cmu
 
Clinical medicine
Clinical medicineClinical medicine
Clinical medicine
 
การอ่านค่า Ekg
การอ่านค่า Ekgการอ่านค่า Ekg
การอ่านค่า Ekg
 
Survivor NT step2 SIRIRAJ book 2
Survivor NT step2 SIRIRAJ book 2Survivor NT step2 SIRIRAJ book 2
Survivor NT step2 SIRIRAJ book 2
 
Nl part ii march 2009
Nl part ii march 2009Nl part ii march 2009
Nl part ii march 2009
 
ประชุมวิชาการ ศิริราช 52
ประชุมวิชาการ ศิริราช 52ประชุมวิชาการ ศิริราช 52
ประชุมวิชาการ ศิริราช 52
 
ประชุมวิชาการ ศิริราช 53
ประชุมวิชาการ ศิริราช 53ประชุมวิชาการ ศิริราช 53
ประชุมวิชาการ ศิริราช 53
 
Mdcu Obstetrics Tutorial
Mdcu Obstetrics TutorialMdcu Obstetrics Tutorial
Mdcu Obstetrics Tutorial
 
Mdcu Comprehensive Cardio
Mdcu Comprehensive CardioMdcu Comprehensive Cardio
Mdcu Comprehensive Cardio
 
Mdcu Neonatology Review
Mdcu Neonatology ReviewMdcu Neonatology Review
Mdcu Neonatology Review
 
Total parenteral nutrition
Total parenteral nutritionTotal parenteral nutrition
Total parenteral nutrition
 
Mdcu Preventive Screening
Mdcu Preventive ScreeningMdcu Preventive Screening
Mdcu Preventive Screening
 
Upper GI bleeding
Upper GI bleedingUpper GI bleeding
Upper GI bleeding
 
Survivor NT step2 SIRIRAJ book 1
Survivor NT step2 SIRIRAJ book 1Survivor NT step2 SIRIRAJ book 1
Survivor NT step2 SIRIRAJ book 1
 
Abnormal pap smear ศิริราช ppt
Abnormal pap smear ศิริราช pptAbnormal pap smear ศิริราช ppt
Abnormal pap smear ศิริราช ppt
 
ศรว 51 By Cmu
ศรว 51 By Cmuศรว 51 By Cmu
ศรว 51 By Cmu
 
Endocrine Med 2010 Step2
Endocrine Med 2010 Step2Endocrine Med 2010 Step2
Endocrine Med 2010 Step2
 
Mdcu Step2 Gen Sx Ii
Mdcu Step2 Gen Sx IiMdcu Step2 Gen Sx Ii
Mdcu Step2 Gen Sx Ii
 
Step3 Tutorial by SWU book2
Step3 Tutorial by SWU book2Step3 Tutorial by SWU book2
Step3 Tutorial by SWU book2
 
Survival for all draft 1 - 1
Survival for all draft 1  - 1Survival for all draft 1  - 1
Survival for all draft 1 - 1
 

Similar to Mdcu Exam Step 2 2010

Nle step 2_2009 si115-116 and nle_step_2_2009 nctms editors cut
Nle step 2_2009 si115-116 and nle_step_2_2009 nctms editors cutNle step 2_2009 si115-116 and nle_step_2_2009 nctms editors cut
Nle step 2_2009 si115-116 and nle_step_2_2009 nctms editors cut
Loveis1able Khumpuangdee
 
ข้อสอบปลายภาค ภาคเรียนที่ 1 ปีการศึกษา 2555 ชั้น ม.5
ข้อสอบปลายภาค ภาคเรียนที่ 1 ปีการศึกษา 2555 ชั้น ม.5ข้อสอบปลายภาค ภาคเรียนที่ 1 ปีการศึกษา 2555 ชั้น ม.5
ข้อสอบปลายภาค ภาคเรียนที่ 1 ปีการศึกษา 2555 ชั้น ม.5
Nattapong Boonpong
 
Nle step 2_2009 si115-116 and nle_step_2_2009 nctms editors cut key
Nle step 2_2009 si115-116 and nle_step_2_2009 nctms editors cut keyNle step 2_2009 si115-116 and nle_step_2_2009 nctms editors cut key
Nle step 2_2009 si115-116 and nle_step_2_2009 nctms editors cut key
Loveis1able Khumpuangdee
 

Similar to Mdcu Exam Step 2 2010 (20)

Nle step 2_2009 si115-116 and nle_step_2_2009 nctms editors cut
Nle step 2_2009 si115-116 and nle_step_2_2009 nctms editors cutNle step 2_2009 si115-116 and nle_step_2_2009 nctms editors cut
Nle step 2_2009 si115-116 and nle_step_2_2009 nctms editors cut
 
Extern conference : Fracture mid shaft humerus
Extern conference : Fracture mid shaft humerusExtern conference : Fracture mid shaft humerus
Extern conference : Fracture mid shaft humerus
 
ข้อสอบปลายภาค ภาคเรียนที่ 1 ปีการศึกษา 2555 ชั้น ม.5
ข้อสอบปลายภาค ภาคเรียนที่ 1 ปีการศึกษา 2555 ชั้น ม.5ข้อสอบปลายภาค ภาคเรียนที่ 1 ปีการศึกษา 2555 ชั้น ม.5
ข้อสอบปลายภาค ภาคเรียนที่ 1 ปีการศึกษา 2555 ชั้น ม.5
 
Noonยศกร รามาฯ
Noonยศกร รามาฯNoonยศกร รามาฯ
Noonยศกร รามาฯ
 
Low Back Pain
Low Back PainLow Back Pain
Low Back Pain
 
Case conference15.11.61
Case conference15.11.61Case conference15.11.61
Case conference15.11.61
 
Nle step 2_2009 si115-116 and nle_step_2_2009 nctms editors cut key
Nle step 2_2009 si115-116 and nle_step_2_2009 nctms editors cut keyNle step 2_2009 si115-116 and nle_step_2_2009 nctms editors cut key
Nle step 2_2009 si115-116 and nle_step_2_2009 nctms editors cut key
 
Case conference ortho
Case conference orthoCase conference ortho
Case conference ortho
 
Motor weakness and Cerebrovascular Disease
Motor weakness and Cerebrovascular DiseaseMotor weakness and Cerebrovascular Disease
Motor weakness and Cerebrovascular Disease
 
Extern orthopedics conference
Extern orthopedics conferenceExtern orthopedics conference
Extern orthopedics conference
 
Fracture distal end radius
Fracture distal end radiusFracture distal end radius
Fracture distal end radius
 
fracture distal end radius
fracture distal end radiusfracture distal end radius
fracture distal end radius
 
Back pain
Back painBack pain
Back pain
 
Herniated nucleus pulposus
Herniated nucleus pulposusHerniated nucleus pulposus
Herniated nucleus pulposus
 
Fracture Clavicle
Fracture ClavicleFracture Clavicle
Fracture Clavicle
 
Extern conference..
Extern conference..Extern conference..
Extern conference..
 
Orthopedic conference
Orthopedic conferenceOrthopedic conference
Orthopedic conference
 
National license 2010 by med tu 16
National license 2010 by med tu 16National license 2010 by med tu 16
National license 2010 by med tu 16
 
Extern conference-orthopedic
Extern conference-orthopedicExtern conference-orthopedic
Extern conference-orthopedic
 
colles' fracture case dicussion
colles' fracture case dicussioncolles' fracture case dicussion
colles' fracture case dicussion
 

More from vora kun

CPR 2010 อ ปริญญา รามา
CPR 2010 อ ปริญญา รามาCPR 2010 อ ปริญญา รามา
CPR 2010 อ ปริญญา รามา
vora kun
 
NT step2 march 53
NT step2 march 53NT step2 march 53
NT step2 march 53
vora kun
 
Thai Osteoporosis guideline 2553
Thai Osteoporosis guideline 2553Thai Osteoporosis guideline 2553
Thai Osteoporosis guideline 2553
vora kun
 
ortho 02 orthopaedic complication & prevention + orthopaedic trauma (practica...
ortho 02 orthopaedic complication & prevention + orthopaedic trauma (practica...ortho 02 orthopaedic complication & prevention + orthopaedic trauma (practica...
ortho 02 orthopaedic complication & prevention + orthopaedic trauma (practica...
vora kun
 
ortho 06 common ortho dis 2 edited 12 mar 10
ortho 06 common ortho dis 2 edited 12 mar 10ortho 06 common ortho dis 2 edited 12 mar 10
ortho 06 common ortho dis 2 edited 12 mar 10
vora kun
 
ortho 05 common rheumatic dx rx
ortho 05 common rheumatic dx rxortho 05 common rheumatic dx rx
ortho 05 common rheumatic dx rx
vora kun
 
ortho 01 management of open fracture-update by kk 31052010
ortho 01 management of open fracture-update by kk 31052010ortho 01 management of open fracture-update by kk 31052010
ortho 01 management of open fracture-update by kk 31052010
vora kun
 
ortho 04 drugs in orthopaedic (principle & common use)
ortho 04 drugs in orthopaedic (principle & common use)ortho 04 drugs in orthopaedic (principle & common use)
ortho 04 drugs in orthopaedic (principle & common use)
vora kun
 
ortho 03 principle of closed reduction in fracture and dislocation
ortho 03 principle of closed reduction in fracture and dislocationortho 03 principle of closed reduction in fracture and dislocation
ortho 03 principle of closed reduction in fracture and dislocation
vora kun
 
ortho 02 orthopaedic complication & prevention + orthopaedic trauma (practica...
ortho 02 orthopaedic complication & prevention + orthopaedic trauma (practica...ortho 02 orthopaedic complication & prevention + orthopaedic trauma (practica...
ortho 02 orthopaedic complication & prevention + orthopaedic trauma (practica...
vora kun
 
SWU CXR interpretation
SWU  CXR interpretationSWU  CXR interpretation
SWU CXR interpretation
vora kun
 
NeuroSx step2 Review
NeuroSx step2 ReviewNeuroSx step2 Review
NeuroSx step2 Review
vora kun
 

More from vora kun (12)

CPR 2010 อ ปริญญา รามา
CPR 2010 อ ปริญญา รามาCPR 2010 อ ปริญญา รามา
CPR 2010 อ ปริญญา รามา
 
NT step2 march 53
NT step2 march 53NT step2 march 53
NT step2 march 53
 
Thai Osteoporosis guideline 2553
Thai Osteoporosis guideline 2553Thai Osteoporosis guideline 2553
Thai Osteoporosis guideline 2553
 
ortho 02 orthopaedic complication & prevention + orthopaedic trauma (practica...
ortho 02 orthopaedic complication & prevention + orthopaedic trauma (practica...ortho 02 orthopaedic complication & prevention + orthopaedic trauma (practica...
ortho 02 orthopaedic complication & prevention + orthopaedic trauma (practica...
 
ortho 06 common ortho dis 2 edited 12 mar 10
ortho 06 common ortho dis 2 edited 12 mar 10ortho 06 common ortho dis 2 edited 12 mar 10
ortho 06 common ortho dis 2 edited 12 mar 10
 
ortho 05 common rheumatic dx rx
ortho 05 common rheumatic dx rxortho 05 common rheumatic dx rx
ortho 05 common rheumatic dx rx
 
ortho 01 management of open fracture-update by kk 31052010
ortho 01 management of open fracture-update by kk 31052010ortho 01 management of open fracture-update by kk 31052010
ortho 01 management of open fracture-update by kk 31052010
 
ortho 04 drugs in orthopaedic (principle & common use)
ortho 04 drugs in orthopaedic (principle & common use)ortho 04 drugs in orthopaedic (principle & common use)
ortho 04 drugs in orthopaedic (principle & common use)
 
ortho 03 principle of closed reduction in fracture and dislocation
ortho 03 principle of closed reduction in fracture and dislocationortho 03 principle of closed reduction in fracture and dislocation
ortho 03 principle of closed reduction in fracture and dislocation
 
ortho 02 orthopaedic complication & prevention + orthopaedic trauma (practica...
ortho 02 orthopaedic complication & prevention + orthopaedic trauma (practica...ortho 02 orthopaedic complication & prevention + orthopaedic trauma (practica...
ortho 02 orthopaedic complication & prevention + orthopaedic trauma (practica...
 
SWU CXR interpretation
SWU  CXR interpretationSWU  CXR interpretation
SWU CXR interpretation
 
NeuroSx step2 Review
NeuroSx step2 ReviewNeuroSx step2 Review
NeuroSx step2 Review
 

Mdcu Exam Step 2 2010

  • 1. MDCU COMPREHENSIVE EXAM 4. stretching of quadriceps muscle STEP 2 5. increase activity with knee 27 กุมภาพันธ 2552 flexion >90 degree 4. A man experienced a low back pain ขอบคุณเพื่อนๆที่ชวยกันจดจํา  after lifting object from the floor. He was ขอบคุณเกดกับพี่จุบที่ชวยเปนเสมียน  diagnosed with protusion of L4-5 disc. Which of the following nerve root will be ORTHO compressed and what symptoms will he 1. What is the mechanism of injury from present with? seat-belt injury? 1. L4 nerve root – weakness of 1. Flexion injury quadriceps muscle 2. Flexion rotation injury 2. L4 nerve root – weakness of 3. Extension injury tibialis anterior 4. Extension distraction injury 3. L5 nerve root – sensory loss at 5. Vertical compression injury 2. What is INCORRECT about flexion heel rotation injury of TLS spine? 4. L5 nerve root – weakness of 1. It’s unstable injury. tibialis anterior 2. Neurological involvement is 5. L5 nerve root – decreased ankle common. jerk 3. It cannot be treated 5. A 25 yr-old man, sudden right back conservatively. 4. Surgery is indicated when there pain radiate to Rt. leg while he was is progressive neurological picking up object from floor. What’s not deficit. appropriate for acute management? 5. Bad prognosis in the presence 1. bed rest of abnormal reflexes. 2. NSAIDs 3. In the case of chronic OA knee. What 3. pelvic traction is NOT appropriate management? 4. lumbosacral support 1. paracetamol 5. transcutaneous electrical nerve 2. ultrasound therapy stimulation 3. stretching of hamstring muscle
  • 2. 6. Postpartum woman in breast-feeding 1. ultrasound therapy period presented with pain and swelling 2. moist heat of Rt. wrist. PE: tender, nodule at radial 3. laser side of Rt. hand. The pain was 4. cold pack aggravated by doing ulnar deviation. 5. hydrotherapy What is the diagnosis? 9. เด็กชายอายุ 5 ป มีอาการเจ็บสะโพก 1. carpal tunnel syndrome ขวามา 3 เดือน เริ่มเดินกะเผลกมา 1 เดือน 2. cubital tunnel syndrome ตรวจรางกาย hip joint มี limit ROM.ขอใด 3. carpal ganglion ตอไปนี้ผิด 4. stenosing tenosynovitis 1. โรคดังกลาวเกิดจาก ischemic 5. rheumatoid arthritis with necrosis ของ femoral head 2. เปนในเด็กชายมากกวาเด็กหญิง rheumatoid nodule 3. พบ unilateral มากกวา bilateral 7. ผูปวยหญิงวัยรุน ปวดขอมือขวา, 2nd Rt. 4. prognosis จะแยกวาถามีอาการ PIP และ Lt. elbow มา 1 สัปดาห ปฏิเสธ ตอน 10 ป ประวัติ Sexual intercourse PV: normal 5. การรักษาทําเพื่อปองกัน deformity ตรวจรางกาย warm, swelling and 10. เด็ก 12 ป มีอาการ severe pain right tenderness at 2nd PIP and Rt. wrist, Lt. thigh, มารดาใหประวัติวา 1 สัปดาหกอน elbow. What is the most appropriate เกิดอุบัติเหตุจากการเลนฟุตบอล ตรวจ รางกายไมมีไข เด็กมีอาการเจ็บที่ตนขาขวา  investigation? ไมยอมขยับขา จะสงตรวจอะไรเพื่อชวยการ 1. Lt. elbow arthrocentesis วินิจฉัย 2. Rt. wrist arthrocentesis 1. ultrasound Rt thigh 3. H/C 2. X-ray hip with Rt thigh 4. cervical swab ER MED AND TRAUMA 8. ผูปวยหญิง 70 ป chronic bilateral knee  11. Which of the following management pain with morning stiffness less than 20 can offer the best chance in treatment of PEA? min for 6 months, PE: bow leg, atrophy 1. Early defibrillation of quadriceps muscle, tender at medial 2. Early administration of atropine side of both knee, full ROM, pain on 3. Early transcutaneous pacing end. What is the most appropriate management?
  • 3. 4. Rapid identification and 3. observe for signs of correction of immediate compartment syndrome correctable cause 4. tetanus toxoid IM and tetanus 12. 30 yr-old man sustained stab chest antitoxin wound at Lt. parasternal border. He was 15. Which is CORRECT about brain restless but can speak fluently. BP edema? 80/60 PR 110/min PE: engorged neck 1. Brain edema is solely due to vein, equal breath sound, no tracheal vasogenic edema. deviation. After resuscitation with 2. Blood brain barrier disruption is Oxygen and RLS, what’s the next the least likely to cause brain management? edema 1. ET intubation with 3. Uptake and degradation of fluid hyperventilation protein by glial cells is one of 2. continue RLS IV the mechanism of vasogenic 3. consider pericardiocentesis and edema. refer to trauma center 4. Mannitol is effective only in the 4. needle thoracocentesis area that the BBB is broken. 13. A man was struck by a lightning 30 16. ผูปวยหญิงอายุ 20 ปมาหองฉุกเฉิน minutes ago. PE reveals burned wound ดวยหอบ เกร็ง หายใจเร็ว มือจีบ ถาทาน at Lt. buttock and Rt. arm, otherwise เปนแพทยเวร จะทําอยางไร WNL. He is clinically stable. What is the 1. ปลอบผูปวยใหใจเย็นๆแลวซัก  next management? ประวัติ 1. Admit for observation 2. เอาถุงกระดาษครอบปากและจมูก 2. colostomy ทันที 3. sigmoidoscopy 3. ให valium 10 mg IV ทันที 4. wound dressing and discharge 4. เจาะ blood gas ทันที 14. A man was bitten by a snake at his 17. ผูปวยประสบอุบติเหตุรถชน มี pelvic ั leg 30 minutes ago. The leg is swollen fracture (APC III) with perineal and erythematous. There is a fang mark laceration. After initial resuscitation, the at the wound. He also has hematuria. subsequent management would consist Which of the following management is of which of the followings? NOT appropriate? 1. ORIF with debridement and 1. coagulogram colostomy 2. antivenum
  • 4. 2. pelvic stabilization and 3. fiberoptic guided nasotracheal colostomy intubation 3. ORIF and debridement 4. cricothyroidotomy 4. explor perineum 5. tracheostomy 5. pelvic traction and debridement 21. ผูปวยอายุ 25 ปประสบอุบตเหตุ ั ิ 18. ผูปวยชายมีอาการปวดทองนอยรุนแรง รถยนต แรกรับที่ ER ขาสองขางไมเทากัน เปนตอนที่กําลังปสสาวะ แรกๆปสสาวะไหล ผูปวยหายใจไดเองปกติ เสียงลมหายใจ ดี จากนั้นปสสาวะหยุดทันที และมีอาการ เทากัน 2 ขาง BP 90/60 mmHg ให RLS ปวดขึ้นมาทันที คิดถึงโรคใดมากที่สุด IV load ไป 2000 ml ปรากฏวา BP ยังคง 1. urethral stone 90/60 อยู ควรทําอยางไรตอไป 2. vesical stone 1. load RLS ตออีก 2000 ml. 3. ureteral stone 2. emergency explor. Lap. 4. acute pyelonephritis 3. ทํา DPL ที่ lower abdomen 5. acute cystitis 4. emergency angiography 19. ผูปวยอายุ 18 ปถกสุนัขกัดมา 1 เดือน ู 5. group O Rh - blood transfusion มีประวัตฉีดบาดทะยักครบ 5 เข็มแลว และ ิ 22. ผูปวยตกตึก 3 ชั้น unconscious ลอง เคยฉีด Rabies vaccine ครบ 5 เข็มมาเมื่อ กดที่ supraorbital ridge ผูปวยไมลืมตา ไม อายุ 8 ป จะให vaccine อยางไร พูด จงบอก GCS 1. Rabies vaccine booster 2 doses 1. 4 2. Rabies vaccine 3 doses 2. 5 3. Rabies vaccine 5 doses 3. 6 4. Rabies vaccine 5 doses and 4. 7 Rabies immunoglobulin 5. 8 5. เจาะเลือดตรวจระดับ Rabies Ab 23. ผูหญิงวัยรุน นําสง ER เนื่องจากหมด ถามากกวา protective level ไม สติ ตรวจรางกายมี pinpoint pupil หายใจ ตองให vaccine ชา ความดันต่ํา ไมมี neurodeficit คิดวา 20. ผูปวยชาย โดนรถชนหมดสติ มีรอยช้ํา เกิดจากอะไร ที่ทายทอยและแผลที่คอดานหลัง ที่ ER มี 1. opioid overdose inspiratory stridor ควรจัดการกับ airway 2. alcohol intoxication ของผูปวยรายนี้อยางไร 3. benzodiazepine overdose 1. head tilt-jaw thrust และ ใช PED AMBU bag 24. A 10-yr old girl visits you because of 2. direct laryngoscope and ET a goiter. Her mother states that she has intubation poor performance at school. PE: diffuse
  • 5. thyroid enlargement, rubbery 2. Serum electrolyte consistency, irregular surface. TFT: 3. USG KUB system FT4=6 FT3=1.4 TSH=20. What is the 4. VCUG best investigation for diagnosis? 5. IVP 1. thyroid globulin 2. USG thyroid 27. 5 yr-old boy was stung by wasps 10 3. FNA days ago. He came to hospital because 4. thyroid scan of fever, swollen wrist and ankle and 5. Antinuclear antibody urticaria. What is the most likely 25. Male term neonate with exclusive diagnosis? breast feeding developed jaundice on 1. serum sickness DOL 3. He was treated with intensive 2. post infective arthritis phototherapy. Serum bilirubin decline from 20 mg/dl to 19 mg/dl in 4 hr. His 3. pyomyositis laboratory test shows that Coomb’s test 4. toxic vasculitis negative, G6PD not deficient. PBS: 5. Henoch-Schonlein purpura microspherocyte. Maternal blood group 28. Male neonate, BW 2000g with is B Rh+. His blood group is O Rh+. dyspnea and cyanosis. PE: meconium What is the next management? staining at nail, grunting and stridor. 1. continue phototherapy, exchange if MB>20 What is the first management? 2. continue phototherapy, 1. give oxygen exchange if MB>25 2. suction at hypopharynx 3. partial exchange transfusion 3. evaluate APGAR 4. total exchange transfusion 4. Intubation 5. stop breast feeding and switch 5. positive pressure ventilation to formula, then repeat MB 29. A 1-hr old male neonate with BW again 26. A full term neonate, PE: penoscrotal 2000 g. BT 35.5°C HR 180/min BP opening of the urethral meatus, cannot 40/28 mmHg. ABG: pH=7.16 PaO2=30 palpate both sides of testis in the PCO2=50 HCO3=12. What is the least scrotum. What is the emergency likely to be the problem in this patient? management? 1. cold stress 1. Serum creatinine 2. respiratory acidosis
  • 6. 3. metabolic acidosis 3. salbutamol NB with ipatropium 4. hypoxemia bromide 5. hypovolemic shock 4. budesonide NB 5. adrenaline NB 30. An 8-month-old girl presents with 2 33. 2 yr-old male presented with calf days of high fever. Physical examination pain. PE: hematoma at calf with reveals BT 38oC, tense anterior tenderness, multiple large ecchymosis fontanelle, clear sensorium, and at lower extremities. Coagulogram generalized maculopapular rash. Which reveals prolonged PT, normal PTT and of the following is the most likely TT. What is the proper management? diagnosis? 1. cryoprecipiate 2. Vit. K 10 mg IV 1. Varicella 3. DDAVP 2. Measles 4. plt. transfusion 3. Exanthem subitum 34. Preterm neonate GA 32 wk BW 4. Erythema infectiosum 1600 develops grunting and retraction 5. Viral meningitis after delivery. What is the appropriate early management? 31. A 6 yr-old boy presented with 1. ABG erythematous vesicle, pruritus and 2. CPAP excoriation at popliteal and cubital area. 3. CXR What is the appropriate management? 4. TPN 1. topical cotrimazole 5. IV fluid 2. topical triamcinolone 35. 6 yr-old boy presented with fever, 3. oral doxycycline barking cough and stridor. PE: stridor, 32. A child presented with acute barking cough, harsh breath sound, asthmatic attack. At ER, he was given 3 inspiratory stridor. Subcostal and doses of NB salbutamol. After supracostal retraction RR 32/min What’s administration of 3 doses, wheezing and the appropriate management? retraction still persist. He still has 1. salbutamol NB respiratory distress. What is the next 2. ipatropium NB management? 3. adrenaline NB 1. IV hydrocortisone 4. IV corticosteroid 2. Intubation 5. ET intubation
  • 7. 36. A 9 yr-old girl was brought to ER 5. primary immune deficiency because of high fever, hematemesis 39. A male neonate, BW 2000g and abdominal pain. PE reveals BT developed dyspnea shortly after delivery. 39.5°C and hepatomegaly. CBC: Hct PE: meconium staining at nail, 45.6% WBC 1500 (N 30% L 65% AL intercostal retraction. What’s the 5%). What is the most likely diagnosis? diagnosis? 1. Septic shock 1. respiratory distress syndrome 2. Dengue hemorrhagic fever 2. meconium aspiration syndrome 3. Viral myocarditis 3. transient tachypnea of newborn 4. Aplastic anemia 40. A 8-yr old boy presented with fever 5. Idiopathic thrombocytopenic and Lt. hemiparesis for 1 wk. BT 39°C purpura with clubbing fingers. Motor weakness 37. What is the most likely to be born to gr.II Lt. side. What’s the treatment? mother with Graves’ disease that is 1. IV antibiotics then observe currently controlled? 2. intrathecal antibiotics then observe 1. hypothyroid infant 3. IV steroid then observe 2. hyperthyroid infant 4. surgical drainage 3. mongoloid infant 5. Emergency craniotomy 4. infertile infant 41. เด็กอายุ 7 ป มีไข ปวดเขามา 2 5. infant with ambiguous genitalia สัปดาห ตรวจรางกาย HR 120/min RR 20/min BP 100/80 BT 38°C CVS: 38. A 5 yr-old boy was brought to the pansystolic murmur gr. II/IV MS: hospital. He has a history of recurrent swelling Rt. knee. What’s the respiratory tract infection and failure to diagnosis? thrive. Physical examination reveals 1. Rheumatic fever chronically ill child, oral thrush and 2. Viral myocarditis insect-bite dermatitis both legs. The 3. Kawasaki disease other history is that both of his parents 4. Rheumatoid arthritis died from pulmonary TB. What’s the 5. Salmonella infective endocarditis most likely diagnosis? 42. A 6 yr-old leukemic boy มีไขมา 5 วัน 1. malnutrition และ bleeding per gum มี purpura ที่ขา 2. disseminated TB 3. disseminated candidiasis BP 70/40 BT 40°C Hct 20% WBC 800 4. pediatric HIV
  • 8. (N5% L 95%) plt. 30000 นึกถึงภาวะใด 2. H2 blocker มากที่สุด 3. UGI study 1. septic shock 4. USG abdomen 2. DHF 5. Endoscope 3. viral meningitis 46. เด็กกัมพูชา 10 ขวบ มาดวยไข เจ็บคอ 4. infective endocarditis ไอ ตรวจรางกายมี dirty patch ที่ tonsil ทั้ง 43. เด็ก 8 ขวบ มีไข ไอ เจ็บคอ หายใจ สองขาง ตับมามไมโต จงใหการวินิจฉัย เหนื่อย ตรวจรางกาย BT 40°C , whitish 1. Diphtheria infection mucopurulent patch at pharyngotonsillar 2. Infectious mononucleosis area both sides, neck swelling, 3. Streptococcal pharyngitis hepatomegaly จงใหการวินิจฉัย 4. Leptospirosis 47. เด็กอายุ 2 ป มาดวย multiple flaccid 1. Diphtheria infection 2. Infectious mononucleosis bullae and superficial erosion at upper 3. Streptococcal pharyngitis lip and chin. ควรตรวจอะไรตอเพื่อการ 4. Leptospirosis วินิจฉัย 5. retropharyngeal abscess 1. HIV Ab 44. เด็กหญิงอายุ 3 ป ซีดมา 1 เดือน แมให 2. Tzanck smear ประวัติวาติดจุกนม กินแตนม ไมยอมกิน 3. biopsy ขาว PBS: hypochromic microcytic 4. KOH anemia จงให management ที่ประหยัด และปลอดภัยที่สุด 5. gram stain 1. ให FeSO4 48. เด็กชายอายุ 6 ป เหลืองมา 3 วัน ซึม 2. Hb typing ลงมา 1 วัน BT 38°C BP ~90mmHg RR 3. เจาะ serum ferritin 20, stupor, hyperreflexia, marked 4. stool occult blood jaundice, no hepatosplenomegaly. What 45. เด็กอายุ 10 ป มาดวยปวดทองมา 2 is the proper management? เดือน wt loss 1 kg จะปวดมากหลังกินขาว 1. lactulose มี awakening pain, N/V ไปพบแพทยได 2. NAC via NG tube antacid, ยาแกปวด อาการไมดีขึ้น จงให 3. D10W 10 cc/kg IV การรักษา 4. intubation and hyperventilation 1. amitryptiline hs 49. เด็กอายุ 1 ขวบ full term BW 3200 g ประวัติแข็งแรงมาตลอด กินเฉพาะ full cow
  • 9. milk 8 oz. 20-30 cc/feed ปริมาณ 6 52. ผูที่แพไขชนิด anaphylaxis ไมควรฉีด feeds/day ไมยอมกินอาหารอื่นๆ ตรวจ วัคซีนปองกันโรคใดตอไปนี้ รางกายพบวาซีด ไมเหลือง ตับมามไมโต 1. JE การเจริญเติบโตปกติดี PBS: hypochromic 2. Influenza microcytic anemia anisocytosis 1+ CBC: 3. H. influenzae Hb 8 MCV 67 RDW 19 other WNL. 4. meningococcal meningitis What is the diagnosis? 5. pneumococcus 1. Iron deficiency anemia 53. ผูปวยเด็กแรกคลอด ตรวจรางกาย 2. Folate deficiency พบวามีลําไสมากองอยูนอกหนาทอง ไมมี 3. B12 deficiency ถุงหุม ขอใดกลาวผิดเกี่ยวกับภาวะดังกลาว 4. Niacin deficiency 1. พบ intestinal atresia ได 10% 5. Vit. C deficiency ของ case 50. ทารกแรกเกิดเพศชาย ไมมี urine ออก 2. สัมพันธกับภาวะ polyhydramnios เลยหลังเกิด 48 ชม. ตรวจรางกายเจอ 3. พบวามี associated anomaly ได bilateral flank masses with tense ascites มาก ทํา urine cath ไมได urine เลย จงใหการ 4. สายสะดือเกาะปกติ วินิจฉัย 54. เด็กอายุ 5 ป มาดวย recurrent UTI, 1. UPJ obstruction VCUG ปรกติ ควรสง investigation ใดตอ 2. VUR 1. IVP 3. cystic nephroma 2. CT scan 4. Wilm’s tumor 3. USG 5. neuroblastoma 55. ทารกแรกคลอดครบกําหนด BW 3200 51. เด็กชายอายุ 15 ป ญาตินําสงเนื่องจาก g คลอดทางชองคลอด ไมมีอาการผิดปกติ นอนหมดสติ ขางตัวมีขวดยาเปลาตกอยู ที่ แตเจาะน้ําตาลได 35 mg/dl ควรทําอยางไร ER no response, GCS 4 bilateral 1. ให IV ทาง umbilical artery pinpoint pupils RR7/min After intubation, 2. ให IV เปน 10%D/W what is the next appropriate 3. ใหกิน 5%D/W ทันที management? 4. ให IV. . . . 1. IV naloxone 56. เด็กชายอายุ 3 ขวบ มาดวย cough 2. IV flumazenil and dyspnea หลังจากมีอาการ URI เมื่อ 3 3. IV atropine วันกอน มี respiratory distress มา 2 วัน 4. gastric lavage ตรวจรางกาย cyanosis, barking cough, 5. activated charcoal via NG tube decreased breath sound both lungs. What’s the next management?
  • 10. 1. ipatropium bromide NB 3. Electrical stimulation for Rt. limb 2. steroid NB 4. ADL training 3. intubation 5. Ambulation training 4. salbutamol NB 60. A man suffered from Lt. hemiplegia 57. เด็ก 8 ป แมพามารพ.เนื่องจากพบวา applied for a rehabilitation program. ซึมลง พบขวดเปลาไมมฉลากตกอยูขางตัว ี After hard work of rehabilitation, he PE: lethargy, pinpoint pupil. Oral and develops Lt. arm edema. What is the nasa cavity: copious secretion. lung: appropriate management? crepitation both lungs. Abdomen: 1. Continued rehabilitation hyperactive bowel sound. จงใหการ 2. Elastic bandage and rest วินิจฉัย 3. Consult radiologist for Doppler 1. opioid overdose ultrasound 2. organophosphate poisoning 3. Phenobarbital overdose 4. Stellate ganglion block 4. paraquat poisoning 61. ผูปวยหญิง อายุ 30 ป ปวดคอและ 58. ผูปวยเด็กอายุ 6 เดือน มี underlying สะบักทั้งสองขาง มี muscle spasm ตรวจ เปน truncus arteriosus มาดวย mild รางกายเจอ trigger point at cyanosis, tachypnea, tachycardia ตรวจ supraspinatus muscle จะแนะนําการออก รางกาย crepitation and hepatomegaly กําลังกายอยางไร CXR: cardiomegaly with increased 1. aerobic exercise pulmonary vasculature What is the appropriate Management? 2. strengthening exercise 1. knee-chest position 3. stretching exercise 2. Furosemide 62. ผูหญิง 25 ป ประสบอุบัติเหตุ MCA 3. Morphine มา 2 เดือนกอน มี spinal cord injury 4. Bicarbonate ระดับ T2 ASIA II ควรวางแผน 5. Beta-blocker rehabilitation program สําหรับ respiratory REHAB system อยางไร 59. What’s the most appropriate 1. Aerobic exercise rehabilitation program for Rt. thalamic 2. Diaphragmatic respiration and hemorrhage patient diagnosed for 2 purse lip breathing days? 3. incentive spirometry 1. Bed positioning 4. Arm swing 2. Active ROM exercise PREVENTIVE
  • 11. 63. ผูปวยชาย 38 ป มีประวัตครอบครัว ิ 4. ปศุสัตว บิดาเปนเสนเลือดหัวใจขาดเลือดเมื่ออายุ 50 ป มาตรวจรางกายประจําป ควรจะสง 67. Influena vaccine ตองใหภายในกี่วัน ตรวจอะไรมากที่สุด กอนการสัมผัสเชื้อจึงจะมีภูมิคมกัน ุ 1. CPK 1. 3 วัน 2. echocardiogram 2. 7 วัน 3. Serum lipid profile 3. 14 วัน 4. EKG 4. 21 วัน 5. Exercise stress test 5. 28 วัน 64. ผูหญิงอายุ 50 ป ไมมีอาการผิดปกติ 68. หากตองการตรวจสอบการระบาดของ มาตรวจรางกาย การตรวจหรือสงตรวจ lab polio ในชุมชน ควรบอกใหทีมสํารวจ อะไรตอไปนี้เหมาะสมนอยที่สุด 1. การตรวจเตานมดวยตนเอง ตรวจสอบอะไร 2. การตรวจทางทวารหนัก 1. ดูแหลงน้ํา 3. pap smear 2. อาหารที่ปนเปอนเชื้อ 4. serum cholesterol 3. สัตวเลี้ยง 5. tumor marker 4. เสื้อผา 65. ขอใดถูกเปนสิ่งที่ “นาทํา” ในการตรวจ 69. การตรวจ stool occult blood สําหรับ คัดกรองในประชากรกลุมเสียง ่ 1. Chronic hepatitis infection – คัดกรองโรคมะเร็งลําไสใหญมี sensitivity ตรวจ ultrasound liver 80% specificity 20% ถาในประชากรมี 2. Chronic hepatitis infection – ความชุกของโรค 10% โอกาสที่จะเปน ตรวจ AFP มะเร็งลําไสใหญเมื่อผลตรวจบวกเปนเทาไร 3. มี risk atherosclerosis – ตรวจ 1. 10% HDL, cholesterol, TG 2. 31% 4. Menopause – ตรวจ bone 3. 50% density 4. 70% 5. 90% 66. ถามี Hepatitis A ระบาดในชุมชน ควร จะให vaccine ปองกันคนกลุมใด 1. ครูโรงเรียนประถม 2. อาสาสมัครสาธารณสุข 3. บุคลากรทางการแพทย
  • 12. STAT 72. ฝการศึกษาหาความสัมพันธของการ 70. นาย A ตองการศึกษาภาวะภูมิแพใน บริโภค caffeine กับการเกิด migraine กลุมเกษตรกร ไดผลดังนี้ ไดผลดังตาราง จงหา ODD ratio จํานวนที่ เปนภูมิแพคดเปน ิ Migraine No ศึกษา (%) disease migraine ฟารม 1000 27% Caffeine - 40 200 เห็ด Caffeine + 60 100 ทํานา 6000 10% 1. 0.2 ฟารม 3000 13% 2. 0.33 เปด 3. 0.6 นาย A สรุปวาการทําฟารมเห็ด เปนปจจัย 4. 2 เสี่ยงตอการเกิดโรคภูมิแพ จงวิเคราะหผล 5. 3 การศึกษา PSYCHI 1. ถูกตอง เพราะ % คนเปนภูมิแพใน 73. ผูปวยหญิง ถูกวิงราวและทําราย ่ อาชีพฟารมเห็ดมีคามากกวา รางกายเมื่อ 2 เดือนกอน ตอจากนั้นมี คาเฉลี่ย อาการกลัว ฝนถึงบอยๆ ไมกลาไปขางนอก 2. ไมถูก เพราะเปน cross sectional ตอนกลางคืน มีอาการ panic attack เปน study บางครั้ง การวินิจฉัยตาม axis I คืออะไร 3. ไมถูก เพราะจํานวนคนของฟารม 1. Acute stress disorder เห็ดนอยกวาฟารมเปด 2. Adjustment disorder 4. ไมถูก เพราะจํานวนคนในแตละ 3. Generalized anxiety disorder กลุมไมเทากัน 4. Posttraumatic stress disorder 71. Incidence ของการเกิด CA lung ใน 5. Panic disorder smoker มีคาเทากับ 6/1000 สวน 74. เด็กอายุ 2 ขวบโดนขัดใจ งอแง ลงไป incidence ในกลุม nonsmoker มีคาเทากับ ดิ้นกับพื้น แพทยควรใหคําแนะนํา 2/1000 จงหา incidence ของการเกิด CA ผูปกครองวาอยางไร lung ที่เปนผลจาก smoking เทานั้น 1. อยาขัดใจ เพราะเด็กอาจอารมณ a. 4% เสีย b. 25% 2. ปลอยไวเฉยๆไมสนใจ ใหหยุดรอง c. 33% เอง 3. ตีใหเจ็บ จะไดไมทําอีก d. 50% 4. ขูวาถาไมหยุดรอง แมจะไมรัก e. 67% 5. อธิบายเหตุผลใหเด็กจนเขาใจ แลว ใหสิ่งที่เด็กตองการ
  • 13. 75. ผูปวยหญิงอาชีพครู มาพบแพทยดวย  3. เด็กไมไดสูบบุหรี่จากการติด เรื่องมีอาการวิตกกังวลอยางมาก กังวลเรื่อง นิโคติน อันตรายนอกบาน กังวลเกี่ยวกับเรื่องลูก 79. ผูชายรูสกกลัวเวลาอยูที่สง กลัวจะโดด ึ  ู และสามี เรื่องความปลอดภัยภายในบาน ตึก เวลาเห็นมีด ก็คิดที่จะนํามีดมาแทง จงใหการวินิจฉัย 1. adjustment disorder ตัวเอง ตนเองไมอยากฆาตัวตาย รูวา  2. OCD ความคิดไมเขาทา แตไมสามารถยับยั้ง 3. PTSD ความคิดตนเองได จงใหการวินิจฉัย 4. generalized anxiety disorder 1. Schizophrenia 76. จงบอกแนวทางการรักษาผูปวยที่มี 2. OCD ความคิดฆาตัวตายที่มีความเสี่ยงสูงมาก 3. Depression 1. นัดตรวจวันรุงขึ้น ยาตานเศรา 4. GAD จิตบําบัด 80. วัยรุนตองการที่จะฆาตัวตาย กาวราว 2. นัดตรวจวันรุงขึ้น ยาคลาย กังวล รักษาดวยไฟฟา ตั้งแตอกหักจากแฟน Dx 3. รับไวในโรงพยาบาล ยาตาน 1. Antisocial เศรา จิตบําบัด 2. Depression in adolescent 4. รับไวในโรงพยาบาล ยาคลาย 3. Adolescent turmoil กังวล จิตบําบัด 81. เด็กอายุ 18 ป มีประวัติไมยอมไป 5. รับไวในโรงพยาบาล ยาตาน โรงเรียน 2 เดือน มีพฤติกรรมกาวราวใสคน เศรา รักษาดวยไฟฟา ในครอบครัว ทํารายรางกายตัวเองกรีดแขน 77. บทบาทของแพทยในการดูแลผูปวยติด เปนรอยทั้งแขน ปนขึ้นหลังคาขูวาจะ  ยาเสพยติดตามพรบ.ฟนฟูสมรรถภาพผูติด ยาเสพยติด จะเริ่มที่ขั้นตอนใด กระโดดตึกฆาตัวตาย พี่สาวมีประวัติรักษา 1. วินิจฉัย กับจิตแพทย จงวินิจฉัย 2. คนหา 1. school refusal 3. หยุดยา 2. adjustment disorder in 4. ฟนฟู adolescent 78. ขอใดผิดเกี่ยวกับ smoking ในผูใหญ 3. depression in adolescent และเด็ก 4. adolescent turmoil 1. ผูใหญตองการนิโคตินในการเลิก 5. schizophrenia นอยกวาเด็ก 2. เด็กตองการมี morning nicotine 82. ขอใดจัดวาเปน psychosis มากกวา 1. ขามถนนกลัวรถชน
  • 14. 2. กลัวคนอื่นมาทํามิดีมิรายจากอาชีพ 85. ชายอายุ 30 ปอาชีพ graphic designer พนักงานตรวจตั๋วโรงหนัง มาดวยอาการปวดทองนอยเปนๆหายๆ 2 3. กาวราว ป ถายลักษณะเหลว ไมมีมูกเลือด รูสึกวา 4. ไดยินเหมือนมีคนเรียกแตไมแนใจ ถายไมสุด ตรวจรางกายปกติ จงวินิจฉัย เลยหันกลับไปดู 1. somatization disorder 83. ผูปวยหญิง 33 ป มาหองฉุกเฉินดวย 2. somatoform pain disorder อาการปวดหัวมาก มีประวัติ admit ดวย 3. somatoform autonomic psychosis 3 ครั้งในชวง 3 ป ประวัติ dysfunction เพิ่มเติมชวง 3 เดือนมานี้ ผูปวยชอบทํา 4. conversion disorder อะไรซ้ําๆ เนนรายละเอียด แตงตัวแปลกไป 5. somatoform disorder unspecified ไมสนใจการเขาสังคม แยกตัว ไมมีเพื่อน 86. ผูปวยหญิงพยายามฆาตัวตาย มีอาการ สนิท พูดถึงแตพอแม จัดวามี personality ของ depression เนื่องจากโดนแฟนทิ้ง แบบใด ผูปวยเติบโตมาในครอบครัวที่แตกแยก 1. paranoid กอนหนานี้มีประวัติเคยพยายามฆาตัวตาย 2. schizoid มาแลวหลายครั้ง ขอใดตอไปนี้ไมใชปจจัย  3. borderline เสี่ยงที่สําคัญของการฆาตัวตายในผูปวย  4. ntisocial รายนี้ 5. schizotypal 1. ครอบครัวแตกแยก 84. ชายวัย 45 ปมีความกังวลวาภรรยามีชู 2. depression นอกใจมา 1 ป ที่ผานมาไมเคยมีหลักฐาน 3. ขาด psychological support หรือเหตุการณใดๆที่บอกวาภรรยาจะ 4. เคยซ้ําชั้นตอนเรียนมัธยม นอกใจ ไมวาญาติๆและภรรยาจะอธิบาย FORENSIC อยางไรก็ไมยอมเชื่อ เคยหึงหวงจนถึงขั้นลง 87. ขอใดตอไปนี้ไมเปนความผิดตาม มือทํารายภรรยา ดานการทํางานยังไป กฎหมายอาญา ทํางานไดตามปกติ เปนหัวหนาแผนกใน 1. การกระทําโดยจําเปนตามมาตรา บริษัทแหงหนึง จงใหการวินิจฉัย ่ 67 1. generalized anxiety disorder 2. การพยายามทําแทงโดยหญิงนั้น 2. delusional disorder ยินยอม 3. การกระทําโดยประมาทเลินเลอไม 3. schizophrenia รายแรง 4. paranoid personality disorder 4. การกระทําโดยบันดาลโทสะ
  • 15. 88. ขอใดตอไปนี้ไมจัดเปนความผิดตาม 2. นาหมอ ตัวทาน กฎหมายอาญามาตรา 276 และ 277 3. นาหมอ เพื่อน 1. นายดําซื้อบริการทางเพศจาก 4. นาหมอ ตัวทาน เพื่อน เด็กชายอายุ 13 ป 5. ไมมีใครผิด เพราะทําเชนนี้กันทุก 2. นางสาวก. อายุ 14 ปมีเพศสัมพันธ คน กับแฟนหนุมอายุ 18 ปอยาง ยินยอมพรอมใจ 91. ปจจัยตอไปนี้มีผลทําใหเกิด rigor 3. นายแดงอายุ 20 ปมีเพศสัมพันธ mortis เร็วขึ้น ยกเวนขอใด กับแฟนสาวอายุ 20 ป แลวแอบ 1. มีไขสูง เสียชีวิตดวย septic shock ถายคลิปไปเผยแพรโดยแฟนสาว 2. จมน้ําและกระเสือกกระสนกอนตา ไมรู 3. CO poisoning 89. ผูปวยหญิงอายุ 16 ป มาพบแพทยดวย 4. ชักกอนตาย เรื่องโดนลุงขมขืนกระทําชําเรา ตรวจ รางกายพบรอยช้ําบริเวณขาหนีบ มีแผลฉีก 5. เสียชีวตในที่อุณหภูมิสูงกวาปกติ ิ ขาดบริเวณอวัยวะเพศ ตองเย็บแผล ผล 92. หากทานลงความเห็นเกี่ยวกับลักษณะ ตรวจ sperm และ acid phosphatase ของบาดแผลโดยสุจริต แตลงความเห็น negative ขอใดถูกเกี่ยวกับการใหความเห็น เกี่ยวกับระยะเวลารักษาหายไมสุจริต ทาน 1. ตรวจพบหลักฐานวา ผูปวยนาจะ   จะมีความผิดตามกฎหมายใดบาง ถูกขมขืนกระทําชําเรา 1. กฎหมายอาญา 2. ตรวจพบหลักฐานวา ผูปวยนาจะ   2. กฎหมายแพง ผานการรวมประเวณี 3. กฎหมายอาญา พรบ.วิชาชีพเวช 3. ตรวจพบหลักฐานวา ผูปวยอาจจะ  กรรม ผานการรวมประเวณี 4. กฎหมายอาญา พรบ.วิชาชีพเวช 4. ตรวจไมพบหลักฐานวา ผูปวยผาน กรรม ขอบังคับแพทยสภา การรวมประเวณี 93. ผูปวยประสบอุบติเหตุ มามแตกตองตัด ั 90. นาของทานเปนแพทยประจํารพ.แหง มามทิ้ง หลังผาตัดนอนโรงพยาบาลอยู 10 หนึ่ง ทานไปหานาที่รพ. เพื่อขอใบรับรอง วันไมมีภาวะแทรกซอน สามารถกลับบาน แพทยไปสมัครเรียนวายน้ํา นาของทาน ได ทานจะลงความเห็นในใบรับรองแพทย เขียนใบรับรองแพทยใหตวทาน และเขียน ั วาอยางไร อีกหลายใบเผือใหเพื่อนๆที่จะสมัครเรียน ่ 1. พักรักษาตัวเปนเวลา 10 วัน หายดี ดวยกัน กรณีเชนนี้ ผูใดมีความผิดบาง 2. ไดรับบาดเจ็บสาหัสเนื่องจาก 1. นาหมอ สูญเสียอวัยวะ
  • 16. 3. อาจเปนอันตรายถึงแกชีวตไดหาก ิ tonsillar exudate both sides, bulging of ไมไดรับการรักษาอยางทันทวงที lateral pharyngeal wall ควรใหการรักษา 4. ไดรับบาดเจ็บตองทนทุกขเวทนา อยางไร 1. aspiration and oral ATB เปนเวลามากกวา 21 วัน จัดวา 2. oral ATB only บาดเจ็บสาหัส 3. IV ATB only 94. โจทยใหรูปหลังของผูปวย มี pattern 4. external drainage and ATB abrasion เปนรูป tram line 5 รอยที่กลาง 5. tonsillectomy หลัง ขอใดผิด 98. ผูปวยอายุ 40 ปมีอาการบานหมุน 1. ถูกตีดวยอาวุธเหลี่ยม มีสันอยาง ขึ้นมาทันทีรวมกับเวียนศีรษะคลื่นไส นอย 5 ที อาเจียนอยางมาก ไมมีเสียงในหู การไดยิน ปกติ อาการบานหมุนไมสัมพันธกับทาทาง 2. ถูกตีดวยอาวุธลักษณะกลมอยาง หรือการเคลื่อนไหวของศีรษะ จงใหการ นอย 5 ที วินิจฉัย 3. ถูกตีดวยอาวุธกลวงอยางนอย 5 ที 1. Meniere disease 4. ถูกตีดวยอาวุธกลมหรือเหลี่ยมก็ได 2. positional vertigo แตไมสามารถบอกจํานวนครั้งได 3. vestibular neuritis 4. paroxysmal nonpositioning ENT vertigo 95. ด็กชาย 4 ขวบ มาดวย greenish 99. เด็กหญิงอายุ 10 ปเปนนักวายน้ําของ discharge from Lt. nostril กินยา ATB มา โรงเรียน มาโรงพยาบาลดวยมี greenish หลายขนานแลวไมหาย ควรทําอะไร nasal discharge และมีอาการปวดที่โหนก 1. ตรวจหา foreign body and แกมและฟนกรามบนดานขวา ตรวจรางกาย remove พบ greenish nasal discharge สอง 2. culture for fungus and treat nasopharynx พบวามี discharge สีเขียว 3. culture and ATB เชนกันและมี mild tenderness over 4. CT sinus maxillary sinus area จงใหการรักษา 96. Organic solvent ที่มีหลักฐานวาทําให 1. topical decongestant only เกิด hearing loss คือสารใด 2. topical decongestant + oral ATB 1. Toluene 3. topical decongestant + oral 2. Styrine antihistamine 3. Xylene 4. topical decongestant + nasal 97. ปวยชาย 25 ปมาดวยไขสง คอบวม อา ู steroid ปากไมขึ้น ตรวจรางกายพบวา มี trismus,
  • 17. 100. เด็กชาย 10 ป เปนหูน้ําหนวกมา 4. IV dobutamine 0.35 mg/min ตั้งแต 8 ป มี discharge กลิ่นเหม็นออกมา 5. IV isoproterenol 5.0 mcg/min เกือบทุกวัน ตรวจ otoscope พบวามี attic 103. A thai male, U/D alcoholism perforation with keratin debris จงใหการ presented with orthopnea, PND, JVP 3 รักษา 1. ATB eardrop cm, S3 gallop, cardiomegaly, pretibial 2. myringoplasty with tube pitting edema, warm extremities. Which 3. modified radical mastoidectomy is the most appropriate management? 4. radical mastoidectomy 1. Lasix 80 mg IV bolus 101. ผูปวยหญิง ประวัติเปน allergic 2. Lasix 300 mg IV drip in 24 hour rhinitis 3 วันกอนมี purulent discharge, 3. Thiamine 100 mg IV nasal congestion, PE: tender both 4. Dobutamine frontal sinus จะสง investigation ใด 1. nasal decongestant 104. A pregnant woman is diagnosed 2. nasal decongestant + amoxicillin with P. falciparum malaria. What is the 3. nasal decongestant + appropriate medication? doxycycline 1. IV quinine 4. nasal decongestant + CT 2. IV artesunate 5. nasal decongestant + 3. Artesunate+mefloquine radiograph 4. Artesunate+doxycycline MED 102. A 48 years old male was admitted 5. Chroloquine+primaquine to coronary care unit with an acute 105. A man has had exertional angina inferior wall myocardial infarction. Two and syncope. His blood chemistry hours after admission, his BP = 86/52 shows total cholesterol 250 and PE HR 40 bpm with sinus rhythm. Which of reveals DBP> 90 on several occasion. the following would be the most What is the diagnosis appropriate initial therapy? 1. Mitral stenosis 1. Immediate insertion of a 2. Mitral regurgitation temporary transvenous 3. Aortic stenosis pacemaker 4. Aortic regurgitation 2. IV atropine sulphate 0.6 mg 5. Tricuspid stenosis 3. IV NSS 300 mL in 15 minutes
  • 18. 106. 70 yr-old woman with underlying 2. bilateral carotid sinus massage disease of DM has watery diarrhea. She 3. IV adenosine said that 5 days ago she had dysuria 4. IV lidocaine and diagnosed to have UTI. 5. synchronized cardioversion Ciprofloxacin was prescribed then. 109. What is the advantage of using Which of the followings has the LEAST colloid solution compared with benefit? crystalloid? 1. sigmoidoscope 1. Colloid solution has bigger 2. stool for C. difficile toxin molecule and doesn’t cross 3. Ba enema plasma membrane. 4. stool occult blood 2. การให colloid ใชอัตราสวน 1:1 5. metronidazole 400 mg bid 7 days เทียบกับเลือด 107. A 50 yr-old woman had chest pain 3. 40 cc of fluid was held in the 3 hrs ago. At ER, her EKG shows plasma for every 1 g of colloid inverted T wave in lead II, III and aVF. infused. Her initial medication includes O2 4. No evidence stated that colloid therapy, IV NTG, beta-blocker and ASA. is better than crystalloid. What is the next proper management? 5. colloid solution is not at risk for 1. IV thrombolytic agent transmitted disease 2. Percutaneous angioplasty 110. A 51 yr-old woman with underlying 3. IV heparin COPD and prolonged intubation for 4. emergency CABG hypercarbic respiratory failure. Her 108. A man came to ER because of current UA shows WBC 3-5/hpf, many palpitation and dizziness. EKG reveals yeast. She is afebrile, clinically stable. supraventricular tachycardia with Foley catheter draining dark yellow ventricular rate of 150 bpm. Right urine. Urine culture grow >10000 carotid sinus massage was done but no colonies of C. albicans. What’s the most response was observed. What is the appropriate management? next management? 1. treat only if U/C grow >100000 1. continue carotid sinus massage colonies
  • 19. 2. treat only if repeat UA show >25 followings is NOT appropriate WBC/hpf management? 3. give fluconazole via NG 1. Insulin IV push 4. off Foley catheter 2. NSS IV load 5. intravesicular amphotericin B 3. IV KCl 111. A 55 yr-old man has substernal 4. IV bicarbonate chest pain for 2 hrs. The pain was dull 5. empirical ATB and radiated to Lt. shoulder but not to 113. A patient sustained spinal cord the arm. At ER, 12-lead EKG was done. injury from T6 level. He came to ER Which of the following management with tachycardia, plethora and should NOT be done? diaphoresis. He claimed that he missed his routine urinary catheterization today. What is the appropriate management? 1. bed rest with leg elevation 2. urinary catheterization 3. stellate ganglion block 114. A 45 yr-old healthy woman has headache for 2 weeks. PE: reveals stiff neck positive. LP was done. CSF open 1. gastric lavage pressure 28 cmH2O, WBC 250 (L>N) 2. IV PPI protein 400, sugar 25 (plasma glucose 3. upper GI endoscopy 100), no bacteria in gram stain. What is 4. Oral Antacid your diagnosis? 5. nutritional support 1. TB meningitis 112. A 75 yr-old female was brought to 2. Aseptic meningitis the hospital because of alteration of 3. Purulent meningitis consciousness. PE reveals fever, 4. Eosinophilic meningitis lethargy, tachypnea. ABG: pH=7.26 5. Cryptococcal meningitis PCO2=56 PaO2=35 HCO3=26. DTx shows hyperglycemia. Which of the
  • 20. 115. Middle aged man came to ER CBC: Hb 7, MCV 102, RDW 20%, because of sudden severe headache, normal WBC and plt. What test should sudden onset of Rt. hemiparesis and be done for the definitive diagnosis? numbness. He was alert at first 1. HAM test presentation but later he lost his 2. Coombs’ test consciousness. PE: right pupil fixed 3. Stool occult blood dilated. After that, he has respiratory 4. G-6-PD assay arrest. What is the most likely 5. Hb typing diagnosis? 118. A patient with underlying disease 1. Superior sagittal sinus of chronic hepatitis C infection had thrombosis percutaneous liver biopsy 3 days ago. 2. Cerebellar hemorrhage Today he develops fever, RUQ pain, 3. Pontine hemorrhage jaundice and melena. What is the most 4. Diffuse subarachnoid likely diagnosis? hemorrhage 1. Hemobilia 5. Medullary hemorrhage 2. Hepatitis from reactivation 116. A man was bitten by a snake with 119. A 20 yr-old woman came to you for black and white stripe in Rayong routine check up. She weighs 97 kg province. At ER, PE reveals fang with 153 cm in height. V/S WNL. PE: marks. Later he developed ptosis, unremarkable. AST 34 ALT 123 HBs Ag dyspnea and muscle weakness. Which and anti-HCV negative. What is the of the following snake has bitten him? diagnosis? 1. cobra 1. HAV infection 2. king cobra 2. HBV infection 3. Malayan krait 3. HCV infection 4. Malayan pit-viper 4. Fatty liver 5. Russel viper 120. A24 yr-old adolescent had sudden 117. 40 yr-old woman developed fatigue onset of pleuritic chest pain and and dyspnea for 3 wks. PE: moderately dyspnea for 1 hour. He is a tall, thin pale, mild jaundice, mild splenomegaly. man with no underlying disease. PE:
  • 21. decrease Rt. breath sound, RR 24/min, this event. Today he presents with normal BP and PR. Chest X-ray shows painful and swollen joint. PE reveals Rt. pneumothorax 50%, otherwise WNL. painful and swollen Rt. knee joint with What is the appropriate management? limited ROM due to pain. His 1. Tube thoracostomy coagulogram shows prolonged APTT, 2. IV fluid normal PT. The laboratory abnormality 3. Consult CVT for video assisted can be corrected by mixing study. And thoracotomy when the patient’s serum is mixed with 4. Observe the serum from hemophilia A patient, it 121. A 65 yr-old man was admitted shows prolonged APTT. What is the because he had post CVA left most appropriate management? hemiparesis. 5 days later, he develops 1. DDAVP Lt. leg edema. PE: left thigh edema, not 2. Cryoprecipitate tender. What is the investigation for 3. Cryo-removed plasma definitive diagnosis? 4. FFP 1. D-dimer 5. PRC transfusion 2. lymphangiography 124. A 60 yr-old man with heavy 3. contrast radiograph smoking, presented with progressive 4. radionucleotide venography dyspnea and dyspnea on exertion. His 5. Doppler USG symptom now is stable. He has 122. Which is the least likely to happen yellowish phlegm 5-6 cc every day. PE: in lightning injury? bilateral rhonchi Hct 49% Which of the 1. apnea following treatment has been showed to 2. cardiac standstill prolong his survival? 3. immediate muscle necrosis 1. ACEI 4. keraunoparalysis 2. theophylline 5. ruptured tympanic membrane 3. Oxygen 123. A 20 yr-old man has a history of 4. Steroid bleeding diathesis since he was a child. 5. beta-agonist No one in the family has experienced
  • 22. 125. A 39 yrs-old male visits your office 4. need surgical correction in bed- for routine check up. His lipid profile ridden case shows cholesterol 150 mg/dl and 5. cardiomyopathy is a clinical triglyceride 300 mg/dl. His father died feature from acute MI 4 yrs ago. What is the 128. 40 yr-old female came to the most appropriate initial management? hospital because she had fever and 1. Dietary modification acute RUQ pain. PE: BT 38°C, 2. Exercise tenderness at RUQ, no rebound 3. Gemfibrozil tenderness. What is the most 4. Cholestyramine appropriate investigation? 5. clofibrate 1. Plain film abdomen 126. A nurse had a history of contacting 2. USG upper abdomen person known to have pulmonary TB. 3. UGI study She came to visit you at your office. 4. CT From her medical record, PPD skin test 5. MRI was done last year with a result of 0.3 129. Which is INCORRECT about DVT? cm at 48 hr. You send her to have PPD 1. Virchow triad consists of skin test again which shows an hypercoagulable state, induration of 13 mm at 48 hr. What endothelial injury and pulmonary should you do next? embolism 1. CXR 130. ชาย 25 ปคัดจมูกตลอดทั้งป ตรวจพบ 2. INH 300 mg/day for 3 months postnasal drip, granular pharynx ไมมี 3. check baseline liver enzymes ประวัติครอบครัวเรื่องภูมิแพ ไมมีประวัติ 4. check liver enzyme every 3 asthma, eczema ตรวจพบ eosinophil months มากใน nasal discharge การตรวจอะไรจะ 5. repeat PPD skin test again next ชวยในการวินจฉัยมากที่สุด ิ week 1. serum IgE level (radiodiffusion) 127. Which is INCORRECT about 2. serum IgE level (radio muscular dystrophy? immunoabsorbent assay) 1. probably muscle enzyme defect 3. food elimination test 2. muscle wasting in specific 4. skin test muscle group 3. genetically determined
  • 23. 131. ผูปวยหญิง 30 ป เปน Graves’ 1. penicillin disease กินยา PTU อยูที่ 150 mg/day มี 2. dicloxacillin maculopapular rash ขึ้นตามตัว ควรทํา 3. ceftriaxone 4. ciprofloxacin อยางไร 5. gentamicin 1. เพิ่ม dose เปน 300 mg/day 135. ผูปวยหญิงอายุ 75 ปญาติพามา 2. ลด dose ลงเปน 100 mg/day โรงพยาบาลดวยอาการซึม ไมคอย 3. เปลี่ยนเปน MMI 7.5 mg/day ตอบสนอง ตรวจรางกาย BP 110/90 PR 4. หยุดยาแลวเปลี่ยนเปน lithium 54/min RR 10/min ซีด ไมเหลือง ฟงไดยิน carbonate เสียงหัวใจเบาลง ไมมี murmur เสียง 5. หยุดยาแลวเปลี่ยนเปน bowel sound ลดลง ไมมี melena เจาะ prednisolone เลือดไดผลดังนี้ 132. สารใดตอไปนี้สัมพันธกับภาวะ CBC: Hb 10.1 WBC 5100 plt. 130000 hyperventilation นอยที่สุด DTx 67 mg/dl 1. methamphetamine E’lyte: Na 126 Cl 109 K 4.5 HCO3 27 2. cocaine ABG: pH 7.23 pCO2 64 pO2 72 HCO3 3. alcohol 27 4. marijuana 133. ผูปวยไตวายเรื้อรัง มาพบแพทย ทานจะสง investigation อะไรเพิ่มเติมเพื่อ การวินิจฉัย แพทยแจงวาจะตองลางไตในอีก 1-2 ป 1. serum cortisol, insulin level, C- ขางหนา ผูปวยไมเคยฉีดวัคซีนใดๆเลย peptide ทานจะแนะนําใหผูปวยฉีดวัคซีนใด ยกเวน 2. TSH, free thyroxine 1. annual influenza virus 3. serum Ca, PTH 2. HAV vaccine 4. EKG, echocardiogram 3. HBV vaccine 5. urine toxicology screen 4. polyvalent pneumococcal 136. ขอใดไมใช hypersensitivity peptide vaccine 1. MP rash + liver enzyme 5. varicella vaccine increase หลังกิน navirapine 2 วัน 134. ผูปวยเบาหวาน ควบคุมระดับน้ําตาล 2. facial angioedema หลังกิน ACEI ไดดี เปน perianal abscess ควรใหยาตัว 3 วัน ใด
  • 24. 3. wheal and flare หลังกิน penicillin characteristics ปกติดี PR มี mass 2 ชั่วโมง anterior to rectum จงใหการวินิจฉัย 4. มี target lesion หลังกิน 1. Vaginal agenesis 2. Imperforated hymen sulfonamide 5 วัน 3. Bicornuate uterus ANES 4. Gonadal agenesis 137. Spinal anesthesia should not be 141. ทา LSA หมายถึงทารกอยูทาใด done in 1. Transverse 1. patient with high fever 2. Cephalic presentation 2. patient with history of post dural 3. Breech presentation puncture headache from 4. blow presentation previous SAB 5. Face presentation 3. patient with mild scoliosis 4. patient with asthma 142. ผูหญิง GA 38 weeks มาดวยอาการ 5. severe pre-eclampsia patient เจ็บครรภ ตรวจพบ deceleration not 138. ยาใดไมควรใหเปน induction agent associated with uterine contraction ในรายที่มี increased ICP ทารกอยูในภาวะใด 1. Thiopental 1. Utero-placental insufficiency 2. Ketamine 2. Vagal stimulation 3. Propofol 3. Oligohydramnios 4. midazolam 5. etomidate 143. หญิงตั้งครรภอายุ 30 ป GA 16 139. A 70 yr-old man has pain after an สัปดาห มาพบแพทย และขอใหแพทยทา ํ operation. What is the appropriate post- การเจาะน้ําคร่ําไปตรวจ chromosome op pain management? เนื่องจากรับไมไดถาลูกจะเปน down 1. paracetamol 500 mg prn syndrome ถาทานเปนแพทยคนนั้น ทาน 2. pethidine 25 mg IM q..h จะทําอยางไร 3. MO 10 mg IM q 6 h 1. ทําตามที่ผูปวยตองการ  4. MO 5 mg IV q 3 h OB-GYN 2. บอกวาความเสี่ยงจากการเจาะมีสูง 140. ผูปวยหญิง 15 ป มาดวยปวดทองทุก กวาความเสี่ยงที่ลูกจะเปน down เดือน (30 วัน) ครั้งหนึงเปน 1-2 วันก็ ่ syndrome จึงไมควรเจาะ หายไป ไมเคยมีประจําเดือนมากอน ตรวจ 3. บอกวาผูปวยกังวลมากเกินไป ไม  รางกายพบวามี secondary sexual ตองเปนหวง
  • 25. 4. บอกวาอายุขนาดผูปวย โอกาสที่ 147. What is the most likely to be found ลูกจะเปน down syndrome มีไม in infant born from mother who drank a เยอะ ใหสบายใจได lot in first trimester? 5. ใหไปพบแพทยคนอื่น 1. Cleft lip, cleft palate 144. Most common cause of uterine 2. Microcephaly rupture between pregnancy is? 3. hydrocephalus 1. การแยกของแผลผาตัด C/S ครั้ง 148. Which of the following ATB is กอน contraindicated in pregnancy? 2. Prolonged labor 1. Ceftriaxone 3. Unattended labor with oxytocin 2. augmentin infusion 3. metronidazole 4. Fetal malposition 4. azithromycin 145. Pregnant woman HIV positive, 5. ofloxacin CD435 ตรวจรางกายพบ Oral candidiasis 149. ผูหญิงทอง GA 32 wk. ปวดทองมา มีอาการไอเรื้อรัง CXR: cavitation AFB 3+ 12 ชั่วโมง เริ่มจาก periumbilical ตอมา which is the least appropriate ยายมาปวดที่ RLQ, PE: guarding at RLQ management จงใหการรักษา 1. ให IRZE 1. Ultrasound 2. Start AZT, 3TC, efavarenz 2. Appendectomy 3. Nevirapine ไมควรใชในผูปวยราย  3. Observe นี้ 4. ATB 4. Plan ที่จะ start ARV โดยเร็วที่สุด 5. Emergency exploratory หลังเริ่ม Anti TB drug laparotomy 146. What is the most common 150. ในผูปวยรายที่มี asymmetrical IUGR symptom of HEELP syndrome? และตรวจพบเปน uteroplacental 1. epigastrium pain insufficiency ทํา Doppler ultrasound จะ 2. headache พบอะไร 3. pulmonary edema 1. Increase umbilical artery 4. blurred vision resistance
  • 26. 2. Decrease umbilical artery 154. หญิงอายุ 55 ป มี abdominal resistance discomfort PE: abdomen mild distension, 3. Increase umbilical artery ascites. USG: suspected left ovarian mass จงบอก chance ที่จะเปน early systemic flow ovarian CA 4. Increase MCA systolic flow 1. 5% 5. Increase MCA resistance 2. 10% 151.GDM class C หลังคลอดตองลด 3. 25% insulin ที่ใหลงเนื่องจากอะไร 4. 50% 1. decrease in human chorionic 5. 75% somatomammotropin EYE 152. ขอใดผิด เกี่ยวกับการขอ consent ทํา 155. ผูปวยเด็ก 5 ขวบ แมพามาพบจักษุ laparoscopic tubal ligation แพทยดวยเรืองตาขางขวาเหลเขาในตั้งแต ่ อายุ 5 เดือน 1. ตอง advice วาจะไมสามารถมีบุตร ตรวจตา VA: RE 20/100 with PH 20/70 ไดอีก LE: 20/30 2. ตองบอกสามีของผูปวยวาภรรยาจะ esotropia 45 prism ไมสามารถมีบุตรได myopia RE +0.5 LE +0.25 3. ตอง advice วาไมรับรองผล 100% no limit of motion ตรวจรางกายอื่นๆอยูในเกณฑปกติ ผูปวย 4. ถาปวดทอง หรือ ไมมีประจําเดือน ไดรับการวินิจฉัยวาเปน esotropia with หลังทํา ตองรีบมาโรงพยาบาล amblyopia RE Management? 5. แนะนําวิธีอื่นในการคุมกําเนิด เชน 1. observe อาจหายไดเองตอนโตขึ้น vasectomy, condom นัด F/U ที่ 1 ป 153. Known case CA ovary มาดวยซึม 2. ใสแวนสายตาตามที่วัดได ตรวจพบ Na 135 K 4 Cl 103 HCO3- 23 3. CT scan เพราะอาจมีความ ผิดปกติของสมองได Ca 18 จงให initial management 4. ปดตาขางที่ดีเพื่อรักษา amblyopia 1. IV NSS keep urine output 200- 5. ผาตัดกลามเนื้อลูกตา 300 mL/hr 156. ผูปวยเด็ก มารดาพามาตรวจดวยเรือง ่ 2. Calcitonin injection ตาเหล วินิจฉัยวาเปน pseudoesotropia 3. bisphosphonate เนื่องจากอะไร 4. Hemodialysis 1. telecanthus
  • 27. 2. epicanthus กวาง protruding at groin region but it can’t be 3. narrow palpebral fissure reduced anymore. He has no fever. The SURG mass is tender. What is the LEAST 157. A 60 yr-old man has reducible right appropriate management? groin mass for 2 years. But 12 hrs ago 1. Try reduction is safe. he developed pain at groin mass. He tried to reduce the mass but found that 2. He needs emergency surgery it is irreducible. He also had nausea because this is an emergency and vomiting. PE: HR 100 BT 38oC. Rt. condition. groin mass 5 cm, irreducible with 3. Elective herniorrhapy can be tenderness. What is the appropriate scheduled. management? 160. Which is correct about papillary 1. Analgesic and muscle relaxant + carcinoma of bladder? trying reduction 1. mostly asymptomatic 2. Analgesic and muscle relaxant + 2. rarely metastasize observe 3. could be treated by TUR-BT 3. Emergency herniorrhaphy ETHICS 4. Emergency laparotomy 158. ผูปวยชาย 70 ป มีอาการ lower 161. ผูปวยเปน end stage lung CA urinary tract symptoms (LUTS) PE: แพทยไดปรึกษาญาติถง prognosis ผูปวย ึ prostate hypertrophy with firm nodular และญาติตัดสินใจกันวา จะไมใสทอชวย หายใจ หลังจากนั้น 2 วัน ผูปวยเกิด  at right lobe. What is the most cardiopulmonary arrest ญาติขอรองให appropriate investigation? แพทยชวยกูชวิตเพื่อรอบุตรสาวกลับมาจาก ี 1. ultrasound KUB ตางจังหวัด แพทยจึงได CPR และใสทอ 2. transrectal ultrasound with ชวยหายใจ วันรุงขึ้นญาติเกิดเปลี่ยนใจ จึง biopsy อยากใหแพทยชวยถอดเครืองชวยหายใจ ่ 3. PSA ออก ขอใดถูกตอง 4. CT scan 1. ไมสามารถทําได เพราะถือเปนการ 159. 50 yr-old man had history of คนตายโดยเจตนา reducible groin mass for 8 years. 2 2. ทําได เพราะผูปวยเคยหยุดหายใจ hours PTA, he noticed a mass ไปครั้งหนึ่งแลว และเปนการทํา ตามที่ผูปวยไดแสดงความจํานงไว 
  • 28. 3. ทําได แตใหญาติเซ็นอนุญาตกอน 164. ผูปวยชาย อายุ 25 ป ใสคอนแทค 4. ทําได เพราะเคยตกลงกับญาติไว เลนสนอน ตื่นเชามาเจ็บตา ไปหาหมอ กอนหนานี้แลว ปรากฎวาเปน corneal abrasion ควรทํา 5. ทําได แตใหญาติเปนฝายถอด อยางไร เครื่องชวยหายใจ 1. หยอดยาชา กลับบาน 162. จากคําประกาศสิทธิผูปวย บิดา 2. topical steroid มารดา หรือผูแทนโดยชอบธรรม อาจใช 3. topical ATB + pressure patch สิทธิแทนผูปวยที่เปนเด็กอายุยังไมเกิน ...  165. ผูปวยหมดสติ EKG เปน Pulseless ป ผูบกพรองทางกายหรือจิต ซึ่งไมสามารถ Electrical activity ควรทําอะไร ใชสิทธิดวยตนเองได 1. Defibrillation 1. 7 ป 2. Atropine 2. 15 ป 3. Adenosine 3. 17 ป 4. Lidocaine 4. 18 ป 166. การศึกษาหนึ่ง แบงกลุมผูเขารวม 5. 20 ป การศึกษาเปน2 กลุมไดแก ผูปวยมะเร็งรัง ADD ON ไข และคนปกติ แลวถามถึงประวัติการกิน 163. เด็กอายุ 6 ป ไขสูง ซึมลงและมีอาการ ยาคุมกําเนิด ถามวาเปนการศึกษาแบบใด ชัก อาศัยอยูแถบชานเมือง ตรวจรางกาย 1. Experimental study ไมรูสึกตัว รูมานตา 4 มิลลิเมตร ตอบสนอง  2. cohort study ตอแสงเล็กนอย คอไมแข็ง รีเฟล็กซ 3. historical cohort study คอนขางไว ผลตรวจน้ําไขสันหลัง ความดัน 4. case control study 180/100 มิลลิเมตรน้ํา เซลล 350 (L 80% 167. ผูปวยมาดวยประวัติ hearing loss N 20%) น้ําตาล 70 (น้ําตาลในเลือด 100) ขางเดียว ตรวจหูพบ Retraction of TM Rt. โปรตีน 50 จงใหการวินิจฉัย ตรวจ Tympanogram ขางขวา type B สวน 1. Toxic encephalopathy ขางที่ปกติ type A ควรทําอยางไร 2. Viral meningitis 1. ตรวจ nasopharynx 3. Purulent meningitis 4. Tuberculous meningitis 5. Fungal meningitis
  • 29. 168. Which of the followings is TRUE 3. Remove blood clot and chemical concerning thyroglossal duct cyst? cautery 1. It was found at lateral of neck 4. Packing 2. Origin ถึง Base of tongue 170. เด็ก 10 ป มาดวย UGIB หลังจาก 169. เด็กชายมาดวยประวัติเลือดกําเดา resuscitation แลว ควรทําอะไรตอไป ออกเปนๆหายๆ มา รพ. เลือดหยุดแลวจะ 1. EGD ทําอะไรตอไป 1. give vaseline ointment 2. Remove blood clot and electrical cautery อะเสวนา ไฮเบญจะ ดอคจุฬา จะพาลานัง บัณฑิตานัง แอคขลัง และเทวัง จะเสวนา พี่หนุม AC มิใชสรณะ โปรดใชวิจารญาณในการอาน Topo MDCU 60 5/3/2552 23.45